Lower Limb (Gray's)

Lakukan tugas rumah & ujian kamu dengan baik sekarang menggunakan Quizwiz!

E. Because the hamstrings cross two joints and are very crucial during all phases of running, but especially during the late swing through midstance phase of running, are easily injured. Their normal action includes hip extension and knee flexion. The do not rotate the hip.

During a 100-meter sprint a 25-year-old male Olympic athlete suddenly pulls up in discomfort and is seen to be clutching the back of his left thigh in agony. Upon further examination the athlete describes the pain as a "tearing" sensation and is unable to flex his knee. Based on these symptoms which of the following actions are affected due to this injury? A. Flexion of the hip and extension of the knee B. Extension of the hip and dorsiflexion C. Medial rotation of the hip D. Lateral rotation of the hip E. Hip extension and knee flexion

D. The fibularis (peroneal) brevis muscle originates from the lateral lower two thirds of the shaft of the fibula and inserts on the tubercle at the base of the fifth metatarsal. Any injury to this area will affect this muscle. Patients will present with a weakness in the eversion of the foot. Fibularis (peroneus) longus, extensor hallucis longus and tibialis anterior all insert on the medial side of the foot and will not be affected in this patient. The gastrocnemius inserts via the Achilles tendon to the posterior surface of the calcaneus.

During a battle, a 19-year-old soldier is shot in the lateral aspect of the right foot by a bullet that ricocheted off a building. The soldier is taken to a field hospital. A radiograph of the foot reveals that the base of the fifth metatarsal was completely obliterated. Which of the following muscles is most likely affected by this injury? A. Tibialis anterior B. Fibularis (peroneus) longus C. Gastrocnemius D. Fibularis (peroneus) brevis E. Extensor hallucis longus

B. This type of injury can result in the "unhappy triad" (of O'Donoghue) injury, with damage to the medial collateral ligament (MCL), anterior cruciate ligament (ACL), and medial meniscus. A blow to the lateral side of the knee stretches and tears the MCL, which is attached to the medial meniscus. The ACL is tensed during knee extension and can tear subsequent to the rupture of the MCL. The remaining answer choices describe structures on the lateral surface of the knee, which are not usually injured by this type of trauma.

During a football game a 21-year-old wide receiver was illegally blocked by a linebacker, who threw himself against the posterolateral aspect of the runner's left knee. As he lay on the ground, the wide receiver grasped his knee in obvious pain. Which of the following structures is frequently subject to injury from this type of force against the knee? A. Fibular collateral ligament B. Anterior cruciate ligament C. Lateral meniscus and posterior cruciate ligament D. Fibular collateral and posterior cruciate ligament E. All the ligaments of the knee will be affected

B. The rough bony projection at the junction of the inferior end of the body of the ischium and its ramus is the large ischial tuberosity. Much of the body's weight rests on these tuberosities when sitting, and it provides the proximal, tendinous attachment of the posterior thigh muscles (hamstring muscles and adductor magnus). The hamstring muscles are associated with hip extension and knee flexion. The adduction of the hip joint will be affected slightly because the adductor magnus is affected, although the rest of the adductor muscles are intact.

During a gymnastic session, a 24-year-old woman suddenly developed pain and swelling on the right buttock. This happened following a forceful thigh movement. There is severe weakness of right hip extension and knee flexion. Adduction of the thigh is also slightly weak. An avulsion fracture of the ischial tuberosity is found on a radiograph. Which of the following group of muscles has most likely involved in this process? A. Adductor brevis, adductor longus, adductor magnus, pectineus, and gracilis B. Biceps femoris, semimembranosus, semitendinosus, and adductor magnus C. Iliacus and psoas major D. Gluteus medius and gluteus minimus E. Gluteus maximus and adductor magnus F. Iliacus, psoas major, rectus femoris and sartorius

B. The gluteal region can be divided into quadrants by two lines positioned using palpable bony landmarks. One line runs inferiorly from the highest point of the iliac crest. The second line runs horizontally midway between the iliac crests and the ischial tuberosity. This divides the gluteal region into four quadrants. The sciatic nerve runs through the lower medial quadrant and must be avoided during intragluteal injections.

During an interview, a 30-year-old man who is a psychiatric patient suddenly becomes aggressive. In order to calm him down, the patient is given an intramuscular injection in the upper lateral quadrant of the buttock. The injection is given at this specific location to prevent damage to which of the following nerves? A. Lateral femoral cutaneous B. Sciatic C. Superior gluteal D. Obturator E. Inferior gluteal

B. The medial branch of the deep fibular (peroneal) nerve accompanies the dorsalis pedis artery and innervates the skin between the contiguous sides of the first and second toes. The saphenous nerve is responsible for cutaneous innervation of the anteromedial aspect of the leg and foot. The superficial fibular (peroneal) nerve innervates most of the dorsum of the foot, with the exception of the area where sensation was lost (medial branch of deep fibular nerve). The common fibular (peroneal) nerve gives off a cutaneous branch, the sural nerve, which innervates the lateral aspect of the leg and lateral side of the foot.

During the preparation of an evening meal a female medical student dropped a sharp, slender kitchen knife. The blade pierced the first web space of her foot, resulting in numbness along adjacent sides of the first and second toes. Which nerve was most likely injured? A. Saphenous B. Deep fibular (peroneal) C. Superficial fibular (peroneal) D. Sural E. Common fibular (peroneal)

E. The posterior cruciate ligament is responsible for preventing the forward sliding of the femur on the tibia. The anterior cruciate ligament prevents posterior displacement of the femur on the tibia. The lateral collateral ligament limits extension and adduction of the leg. The medial meniscus acts as a shock absorber and cushions the articular surfaces of the knee joint.

Following an injury suffered in a soccer match, a 32-year-old woman is examined in a seated position in the orthopedic clinic. Holding the right tibia with both hands, the clinician can press the tibia backward under the distal part of her femur. The left tibia cannot be displaced in this way. Which structure was most likely damaged in the right knee? A. Anterior cruciate ligament B. Lateral collateral ligament C. Medial collateral ligament D. Medial meniscus E. Posterior cruciate ligament

B. The posterior femoral cutaneous nerve arises from nerves S1 to S3. It provides inferior cluneal branches to the lower portion of the gluteal region and a perineal branch to the perineum and supplies sensation to the posterior thigh to the level of the popliteal fossa. Superior gluteal innervation arises from dorsal rami of L1 to L3. Meralgia paresthetica is the occurrence of pain or burning sensations on the lateral thigh, from compression of the lateral femoral cutaneous nerve. The sural nerve, sensory to the lower calf and lateral foot, arises from contributions from the tibial nerve and common fibular (peroneal) nerve. The posterior femoral cutaneous is a sensory nerve and does not innervate muscles.

Following the insertion of a prosthetic hip joint in a 72-year-old man, it was observed that the patient had greatly diminished sensation in the region of distribution of the posterior femoral cutaneous nerve. Which of the following is characteristic of this nerve? A. Cutaneous supply of the superior aspect of the gluteal region B. Arises from sacral spinal nerve levels S1, S2, S3 C. Motor innervation of the obturator internus and gemelli muscles D. Injury results in meralgia paresthetica E. Provides origin of the sural nerve

A. The Achilles tendon inserts upon the calcaneus bone. This tendon represents a combination of the tendons of gastrocnemius and soleus muscles. The tendon of the plantaris can insert with this tendon.

In an accident during cleanup of an old residential area of the city, the Achilles tendon of a 32-year-old worker was cut through by the blade of a brush cutter. The patient is admitted to the hospital and a laceration of the Achilles tendon is diagnosed. Which of the following bones serves as an insertion for the Achilles tendon? A. Calcaneus B. Fibula C. Cuboid D. Talus E. Navicular

B. The iliopsoas forms the lateral portion of the troughlike floor of the femoral triangle. The pectineus forms the medial portion of this floor. The adductor longus provides a medial border for the femoral triangle and meets the sartorius, the lateral border of the triangle, at the apex. The rectus femoris is a superficial contributor to the quadriceps femoris, lateral to the femoral triangle.

In preparing to isolate the proximal portion of the femoral artery, the vascular surgeon gently separated it from surrounding tissues. Posterior to the femoral sheath, what muscle forms the lateral portion of the floor of the femoral triangle? A. Adductor longus B. Iliopsoas C. Sartorius D. Pectineus E. Rectus femoris

D. The tendinous distal portion of the adductor magnus inserts on the adductor tubercle on the upper border of the medial condyle of the femur. The femoral artery passes through the adductor hiatus proximal to this tendinous band, continuing as the popliteal artery. The semimembranosus inserts on the proximal, posterior portion of the tibia. The gracilis inserts with the pes anserinus on the proximal, medial aspect of the tibia. The popliteus inserts on the distal lateral portion of the femur, just above the origin of the lateral head of gastrocnemius. The vastus medialis inserts with other quadriceps muscle components on the patella and then on to the tibial tuberosity.

In the radiographs of the knee of a male 28-year-old basketball player, who had apparently suffered a tear in a medial ligament of the knee, the tubercle on the superior aspect of the medial femoral condyle could be seen more clearly than in most individuals. What muscle attaches to this tubercle? A. Semimembranosus B. Gracilis C. Popliteus D. Adductor magnus E. Vastus medialis

E. The popliteal artery is the continuation of the femoral artery after it passes through the hiatus of the adductor magnus. The popliteal artery divides into the anterior and posterior tibial arteries. The anterior tibial artery passes between the tibia and fibula proximally in the posterior compartment of the leg, whereas the posterior tibial artery continues in the posterior compartment of the leg, to its division into medial and lateral plantar arteries. The posterior tibial artery provides origin for the fibular (peroneal) artery, which supplies the lateral compartment of the leg. The deep femoral artery provides origin for the three or four perforating branches that supply the posterior compartment of the thigh. GAS 600, 617; N 509;

Lower limb angiography of an 82-year-old woman reveals a possible cause for her limb pain during her workout routines in the health spa. The artery that was occluded is one that should have been demonstrable passing between the proximal part of the space between the tibia and fibula. Which of the following arteries is most likely affected? A. Deep femoral B. Popliteal C. Posterior tibial D. Fibular (peroneal) E. Anterior tibial

A. The perineal cutaneous branch of the posterior femoral cutaneous nerve provides a significant portion of the cutaneous innervation of the perineum in some individuals and can require separate anesthetic blockade in childbirth or perineal surgery, if other types of anesthesia are not used. The inferior cluneal branches of the posterior femoral cutaneous nerve supply the lower part of the gluteal skin. The lateral cutaneous branch of the iliohypogastric nerve provides sensation for the anterior superior aspect of the gluteal area. The inferior gluteal nerve innervates the gluteus maximus muscle. The middle cluneal nerves arise from the dorsal primary rami of S1 to S3 and supply skin over the middle of the gluteal region.

The baby was quite large, and the pelvis of the mother-to-be was somewhat narrow, causing her considerable difficulty and pain during the delivery. At her specific request, it was decided to inject local anesthetic into the perineum. The genitofemoral and ilioinguinal nerves were infiltrated anteriorly, and a deep injection was made medial to the ischial tuberosity to anesthetize the pudendal nerve, which supplies much of the perineum in most cases. A few minutes later, it became very obvious to those in attendance that the injection had not been effective enough in the central and posterior parts of the perineum. A separate injection was therefore inserted lateral to the ischial tuberosity. What other nerve(s) can provide much of the sensory supply to the perineum in some individuals? A. Posterior femoral cutaneous B. Inferior cluneal nerves C. Iliohypogastric nerve D. Inferior gluteal nerve E. Middle cluneal nerves

C. The tensor fasciae latae (which is innervated by the superior gluteal nerve) and the iliotibial tract are dense, wide aponeurosis that receives the insertion of the tensor fasciae latae and about 75% of the gluteus maximus. The gluteus maximus is the only one of the muscles listed that is supplied by the inferior gluteal nerve; in fact, it is the only muscle innervated by the inferior gluteal nerve. Gluteus medius and minimus insert on the greater trochanter and are innervated by the superior gluteal nerve. The rectus femoris, supplied by the femoral nerve, inserts via the quadriceps tendon on the patella and tibial tuberosity.

The neurosurgeon had removed a portion of the dense tissue (dura mater) covering the brain of the patient when she removed the tumor that had invaded the skull. To replace this important tissue covering of the brain, she took a band of the aponeurotic tissue of the lateral aspect of the thigh, covering the vastus lateralis muscle. What muscle, supplied by the inferior gluteal nerve, inserts into this band of dense tissue as part of its insertion? A. Gluteus medius B. Gluteus minimus C. Gluteus maximus D. Tensor fasciae latae E. Rectus femoris

B. The apex of the femoral triangle occurs at the junction of the adductor longus and sartorius muscles. The subsartorial (Hunter's) canal begins at this location. Immediately deep to this anatomic point lie the femoral artery, femoral vein, deep femoral artery, and deep femoral vein, often overlying one another in that sequence. This has historically been a site of injuries caused by slipping while handling a very sharp butcher's knife. For this reason, injuries at this location are referred to as the "butcher's block" injury. Fatal loss of blood can occur in just a few minutes if pressure, or a tourniquet, is not applied immediately. The common iliac artery becomes the femoral artery at the inguinal ligament. The saphenous vein joins the femoral vein at the saphenous hiatus, or fossa ovalis. The medial circumflex femoral usually arises from the deep femoral artery about 3 to 5 inches inferior to the inguinal ligament, near the origin of the deep femoral artery from the common femoral. Serious blood loss can occur with injury to any of these vessels, although injury to them is not often fatal.

The news reported that the 58-year-old ambassador received a slashing wound to the medial thigh and died from exsanguination in less than 2 minutes. What was the most likely nature of his injury? A. The femoral artery was cut at the inguinal ligament B. A vessel or vessels were injured at the apex of the femoral triangle C. The femoral vein was transected at its junction with the saphenous vein D. The medial circumflex femoral was severed at its origin E. The deep femoral artery was divided at its origin

A. The patellar ligament is a very heavy ligament that connects the patella to the tibial tuberosity; it provides the insertion of the quadriceps femoris tendon upon the tibia. The patella can be thought of as a bone (a sesamoid bone) that develops within the tendon of the quadriceps femoris muscle. When the reflex hammer strikes the patellar ligament, it stretches the ligament slightly for a brief time, resulting in reflex contraction of the quadriceps femoris muscles. This reflex arc is elicited by the femoral nerve (L4 sensory input component and L2, L3 motor output). The quadriceps femoris includes the rectus femoris and the vastus lateralis, intermedius, and medialis. The patella is the largest sesamoid bone in the body. A sesamoid bone is a bone that develops within a tendon. The quadratus femoris muscle of the gluteal area arises from the ischial tuberosity and inserts on the femur proximally. The sartorius arises from the anterior superior iliac spine and inserts on the proximal, medial aspect of the tibia as one of the three tendinous components of the pes anserinus (goose foot). The biceps femoris of the posterior thigh has a long head that arises from the ischial tuberosity and a short head that arises from the femur; they insert on the head of the fibula.

The patellar reflex appears to be markedly reduced in a 33-year-old diabetic female patient, due to deficient vascular supply of the nerves of her lower limb. The tendon of which of the following muscles is stretched during the patellar reflex? A. Quadriceps femoris B. Quadratus femoris C. Sartorius D. Pectineus E. Biceps femoris

A: Only the femoral nerve innervates the quadriceps femoris. The sciatic nerve, specifically the tibial part, supplies the posterior thigh muscles not the anterior compartment. The obturator nerve supplies the medial compartment and the saphenous nerve does not supply any thigh muscles.

The quadriceps femoris muscle group is formed by the rectus femoris, vastus lateralis, vastus medialis, and vastus intermedius. Which of the following nerves innervates this group of muscles? A. Sciatic nerve B. Femoral nerve C. Obturator nerve D. Saphenous nerve E. Tibial nerve

D. The tibialis posterior tendon is the most anterior of the structures that pass under the laciniate ligament (flexor retinaculum) on the medial side of the ankle to enter the sole of the foot. Increases of pressure within the tissues of the plantar aspect of the foot, usually due to increased fluid from hemorrhage, inflammatory processes, or infections, cause tarsal tunnel syndrome, comparable to carpal tunnel syndrome of the hand. The plantar aponeurosis and other fibrous and osseous tissues of the plantar surface cause this area to be relatively nondistensible; therefore, it takes little increase of fluid content to result in pressures adequate to restrict venous drainage and, thereafter, arterial inflow to the region. Fasciotomy of the medial skin and fascia of the foot and the posterior compartment of the leg can be required to reduce the pressure and allow healing to take place. The structures that pass beneath the flexor retinaculum are, from anterior to posterior: Tendon of tibialis posterior; tendon of flexor Digitorum longus; posterior tibial Vessels and Nerve; tendon of flexor Hallucis longus. (This is the basis of the mnemonic: "Tom, Dick, and a Very Nervous Harry.") Neither the plantaris tendon nor the tibialis anterior tendon pass through this canal.

The swollen and painful left foot of a 23-year-old female long distance runner is examined in the university orthopedic clinic. She states that she stepped on an unseen sharp object while running through the park several days earlier. Emergency surgery is ordered to deal with her tarsal tunnel syndrome. The tarsal tunnel is occupied normally by tendons, vessels, and nerves that pass beneath a very strong band of tissue (flexor retinaculum) on the medial side of the ankle. What is the most anterior of the structures that pass through this tunnel? A. Flexor hallucis longus tendon B. Plantaris tendon C. Tibialis anterior tendon D. Tibialis posterior tendon E. Tibial nerve

A. The superior gluteal nerve innervates the gluteus medius, gluteus minimus, and tensor fasciae latae muscles. The tensor fasciae latae arises from the iliac crest, inserts into the iliotibial tract of the lateral aspect of the thigh, and assists in flexion of the hip. The rectus femoris is innervated by the femoral nerve; it flexes the hip and extends the knee, thus acting upon two major joints. It arises in part from the anterior inferior iliac spine and the rim of the acetabulum and inserts into the quadriceps tendon. The gluteus maximus is supplied by the inferior gluteal nerve. The piriformis and quadratus femoris are both short lateral rotators of the hip and are supplied by branches of the sacral plexus.

Three years following a 62-year-old's hip replacement, the man's CT scans indicated that two of his larger hip muscles had been replaced by adipose tissue. The opinion is offered that his superior gluteal nerve could have been injured during the replacement procedure, and the muscles supplied by that nerve had atrophied and been replaced by fat. Which of the following muscles receives its innervation from the superior gluteal nerve? A. Tensor fasciae latae B. Rectus femoris C. Gluteus maximus D. Piriformis E. Quadratus femoris

B. The common fibular (peroneal) nerve winds around the neck of the fibula before dividing into superficial and deep branches that go on to innervate the lateral and anterior compartments of the leg, respectively. These compartments are responsible for dorsiflexion and eversion of the foot, and injury to these nerves would result in deficits in these movements. The tibial nerve lies superficially in the popliteal fossa. This nerve innervates the posterior compartment of the leg, so compression in this area would result in a loss of plantar flexion and weakness of inversion. The lateral compartment of the leg is innervated by the superficial fibular (peroneal) nerve and is mainly involved in eversion of the foot. The cutaneous branches of the superficial fibular (peroneal) nerve emerge through the deep fascia in the anterolateral aspect of the leg and supply the dorsum of the foot. The anterior compartment of the leg is innervated by the deep fibular (peroneal) nerve and is mainly involved in dorsiflexion of the foot. The medial malleolus is an inferiorly directed projection from the medial side of the distal end of the tibia. The tibial nerve runs near the groove behind the medial malleolus, and compression at this location would result in loss of toe flexion, adduction, abduction, and abduction of the great toe

Upon removal of a knee-high leg cast, a 15-year-old boy complains of numbness of the dorsum of his right foot and inability to dorsiflex and evert his foot. Which is the most probable site of the nerve compression that resulted in these symptoms? A. Popliteal fossa B. Neck of the fibula C. Lateral compartment of the leg D. Anterior compartment of the leg E. Medial malleolus

C. Herniation of the intervertebral disc at L4-5 results typically in compression of the L5 spinal nerve. The L4 spinal nerve exits at the L4-5 intervertebral foramen, but the L5 spinal nerve is put under tension as it passes the herniation to reach the L5-1 foramen. Piriformis entrapment of the fibular (peroneal) division of the sciatic nerve is relatively common, but the dermatome affected here appears to be confined to the L5 distribution to the skin of the foot and also includes the superior gluteal nerve, which supplies the large hip abductors. S1 would involve loss of sensation on the lateral side of the foot and potential weakness in hip extension and plantar flexion. A posterior dislocation of the hip would be unlikely in this injury but, even so, would not result in these deficits.

When he attempted to lift one side of his new electric automobile from the ground to demonstrate his strength, a 51-year-old man felt a sharp pain in his back and quickly dropped the vehicle. Upon examination, it is observed that the patient has deficits in sensation on the dorsum and sole of his foot and marked weakness in abduction and lateral rotation of the lower limb. What was the nature of his injury? A. Piriformis syndrome, with entrapment of the sciatic nerve B. Disc lesion at L3-4 C. Disc lesion at L4-5 D. Disc lesion at L5-S1 E. Posterior hip dislocation

C. The deep fibular (peroneal) nerve supplies the fibularis (peroneus) tertius muscle. Although its name might lead one to think that this muscle is in the lateral compartment with the other two fibularis (peroneus) muscles, it is in the anterior (extensor) compartment of the leg. It is named for its origin from the fibula. It inserts upon the dorsum of the base of the fifth (or fourth) metatarsal bone and assists in extension and eversion of the foot. The sural nerve is a cutaneous nerve, formed by contributions from the tibial and common fibular (peroneal) nerves; it supplies the posterior lateral leg and the lateral side of the foot. The lateral plantar nerve is a branch of the tibial nerve; it innervates the quadratus plantae, muscles of the little toe, the adductor hallucis, lumbricals 2 to 4, and all of the interossei. It is sensory to the lateral side of the sole and the lateral three and a half digits. The superficial fibular (peroneal) nerve supplies the fibularis (peroneus) longus and brevis and innervates the skin on most of the dorsum of the foot. The tibial nerve supplies the calf muscles and divides into the medial and lateral plantar nerves.

Young parents were concerned that their 14-month-old daughter had not yet begun walking. Their pediatrician reassured them, saying that one of the muscles of the leg, the fibularis (peroneus) tertius, had to complete its central neurologic development before the child could lift the outer corner of the foot and walk without stumbling over her toes. What is the most common nerve supply of this muscle? A. Sural B. Lateral plantar C. Deep fibular (peroneal) D. Superficial fibular (peroneal) E. Tibial

D. Osgood-Schlatter disease is also called tibial tuberosity apophysitis and affects the area of the tibial tuberosity. It is not a disease but a problem of overuse, typically in boys of 12 to 14 years or girls 10 to 12 years of age. Very active boys and girls, usually during a growth spurt, are subject to the pain and swelling that occur at the site of attachment of the patellar ligament. The ligament can tear, resulting in a long period of healing following treatment. The medial femoral condyle is the area of attachment of the medial collateral ligament and medial meniscus of the knee joint. The posterior intercondylar eminence is the location of origin of the posterior cruciate ligament. The intercondylar eminence is a bony protuberance on the tibial plateau to which the cruciate ligaments and menisci are attached. The anterolateral tibial tubercle, or Gerdy's tubercle, is the attachment of the iliotibial band or tract; thus it connects the femur and tibia laterally.

A 10-year-old girl is admitted to the emergency department after falling from a tree in which she was playing with her friends. Radiologic and physical examinations reveal Osgood-Schlatter disease (Fig. 5-4). Which of the following bony structures is chiefly affected? A. Medial condyle of tibia B. Posterior intercondylar area C. Intercondylar eminence D. Tibial tuberosity E. Anterolateral tibial tubercle (Gerdy's tubercle)

C. The lymphatic drainage of the foot follows its venous drainage. The small saphenous vein drains the lateral side of the foot and the posterolateral leg. It drains into the popliteal vein in the popliteal fossa. Therefore a lesion on the lateral side of the foot will drain to the popliteal nodes in the popliteal fossa.

A 12-year-old boy is brought to the physician by his father because of redness and swelling of his left foot for 24 hours. Three days earlier he had scraped his foot while wading in a drainage ditch. Examination of the foot shows a purulent abrasion with edema, erythema, and tenderness on the lateral side. Infection will most likely spread from the lateral side of the foot to the regional lymph nodes in which area? A. Lateral surface of the thigh B. Medial malleolus, posteriorly C. Popliteal fossa D. Sole of the foot E. Superficial inguinal area

A. Knowledge of Hilton's Law would lead to this correct answer. This law in a modified form, can be remembered as "a joint is innervated by the same nerves that innervate the muscles that move that joint." A complete explanation of this law can be found in an article by Hebert-Blouin et al., Clinical Anatomy 27:548-555, 2013. The deep fibular (peroneal) nerve is the only nerve listed that innervates muscles that move the ankle joint.

A 15-year-old boy falls and injures his ankle while skateboarding. Examination in the emergency department leads to the conclusion that the ankle is mildly sprained, and it is wrapped with an elastic bandage. The boy still complains of pain in his ankle. Which of the following peripheral nerves is involved in carrying pain sensation from the ankle? A. Deep fibular (peroneal) B. Femoral C. Obturator D. Posterior femoral cutaneous E. Sural

B. The deep fibular (peroneal) nerve is a branch of the common fibular (peroneal) nerve. It is mainly a motor nerve that innervates the anterior compartment of the leg. Its only cutaneous innervation is to the skin of the first web space. The saphenous nerve innervates the medial side of the leg and foot. The cutaneous branch of superficial fibular (peroneal) nerve innervates the anterior part of the lower leg and the dorsum of the foot. The sural nerve innervates the lateral side of the leg and foot.

A 16-year-old boy presents to the emergency department with a fracture of the first and second toes of his right foot. He received an anesthetic injection in the first web space of his foot, to permit easy manipulation and correction. Which nerve was blocked by the anesthesia? A. Saphenous B. Cutaneous branch of deep fibular (peroneal) C. Cutaneous branch of superficial fibular (peroneal) D. Sural E. Common fibular (peroneal)

A. The popliteal lymph nodes are the first to receive lymph from the foot. These nodes will then drain into the deep inguinal nodes and then to the external iliac nodes. The superficial inguinal and internal iliac nodes do not receive lymph from the foot

A 16-year-old boy received a superficial cut on the lateral side of his foot while playing football and is admitted to the emergency department where the wound is sutured. Four days later the patient returns to the hospital with high fever and swollen lymph nodes. Which group of nodes will first receive lymph from the infected wound? A. Popliteal B. Vertical group of superficial inguinal C. Deep inguinal D. Horizontal group of superficial inguinal E. Internal iliac

B. Ligaments act to prevent excessive movement of joints. When a joint is forced into a position, that ligament is stretched and will be tender or rupture if the force is severe enough. Inversion is when the sole of the foot is turned medially and therefore will stretch ligaments that oppose this action. The calcaneofibular ligament is on the lateral side and stretches between the fibula and the calcaneous. It is the only ligament that would be damaged during such an action. The calcaneonavicular and long and short plantar ligaments are located on the plantar surface of the foot and will not be damaged during inversion injuries. The deltoid ligament is located medially and will not be affected.

A 16-year-old teenage girl suffered an inversion sprain of her ankle during dance class. Physical examination in the clinic most likely reveals severe tenderness over which ligament? A. Calcaneonavicular (spring) B. Calcaneofibular C. Long plantar D. Short plantar E. Deltoid

A. The medial meniscus is firmly attached to the medial (tibial) collateral ligament. Damage to the medial collateral ligament often causes concomitant damage to the medial meniscus because of this relationship. The anterior cruciate ligament lies inside the knee joint capsule but outside the synovial cavity. It is taut during extension of the knee and may be torn when the knee is hyperextended. If this were damaged along with the medial meniscus and medial cruciate ligament, an "unhappy triad" (of O'Donoghue, or Donahue, both spellings are correct; also called a "blow knee") injury would result. The lateral meniscus is not attached to the medial collateral ligament but receives muscular attachment to the popliteus muscle. The posterior cruciate ligament also lies outside of the synovial cavity and limits hyperflexion of the knee. The tendon of the semitendinosus forms one third of the pes anserinus, with the tendons of the sartorius and gracilis making up the other two thirds. The pes anserinus (goose foot) is located at the medial border of the tibial tuberosity, and a portion can be used for surgical repair of the anterior cruciate ligament.

A 19-year-old football player was hit on the lateral side of his knee just as he put that foot on the ground. Unable to walk without assistance, he is taken to the hospital. An MRI examination reveals a torn medial collateral ligament. Which structure would most likely also be injured due to its attachment to this ligament? A. Medial meniscus B. Anterior cruciate ligament C. Lateral meniscus D. Posterior cruciate ligament E. Tendon of the semitendinosus

A. With sufficient downward force, the head of the talus can break through the plantar calcaneonavicular (spring) ligament, causing the medial longitudinal arch of the foot to fall, forcing the anterior part of the foot into abduction. The plantar calcaneonavicular ligament is attached between the sustentaculum tali of the calcaneus and the medial surface of the navicular bone, with the head of the talus lying directly upon the inner surface of the ligament. The cuboid bone is located lateral and anterior to the talus bone and would not be fractured. The plantar aponeurosis, a dense, wide band of tissue beneath the fascia of the sole, attaches to the calcaneus and ends distally in longitudinal bands to each of the toes. It stretches very little, even under very heavy loads, and would not rupture in this case. The anterior talofibular ligament is very often injured in "sprained ankle" but would not be directly involved here. The distal tibiofibular joint is a fibrous (and usually nonsynovial) type of joint (called a syndesmosis) between the tibia and fibula, not involved in the displacement of the talus bone.

A 19-year-old patient is admitted to the orthopedic service with a complaint of severe pain in his very swollen and discolored foot. He states that he hurt the foot when jumping from his girlfriend's bedroom window to the concrete driveway below. Plain film radiographic studies reveal that the head of the talus has become displaced inferiorly, thereby causing the medial longitudinal arch of the foot to fall. What would be the most likely, serious problem in such a case? A. Tearing of the plantar calcaneonavicular (spring) ligament B. Fracture of the cuboid bone C. Interruption of the plantar aponeurosis D. Sprain of the anterior talofibular ligament E. Disruption of the distal tibiofibular ligament

E. The calcaneofibular ligament is located inferior and just anterior to the lateral malleolus and connects the lateral malleolus to the calcaneus. The interosseous ligament between the tibia and the fibula is located medially and superior to the lateral malleolus. The tibionavicular ligaments are located on the medial side of the ankle joint, and the point of injury and tenderness is at the lateral side. The anterior tibiofibular ligament is located anterior to the ankle joint, away from the point of injury.

A 19-year-old woman is admitted to the emergency department with complaints of pain and swelling to the right ankle. In a recent volleyball game she jumped to spike the ball then landed on the opponent's shoe with her right foot. She recalls hearing a loud "pop" and felt immediate pain to the ankle. She was unable to bear weight subsequently. On examination, the right ankle was swollen, with maximal tenderness inferior and anterior to the lateral malleolus. Radiographs of the ankle showed no fractures. Which of the following structures were most likely injured? A. Posterior talofibular ligament B. Interosseous ligament between the tibia and fibula C. Tibionavicular ligament D. Anterior tibiofibular ligament E. Calcaneofibular ligament

B. The tensor fasciae latae assists in flexion of the thigh, as well as medial rotation and abduction. Damage to this muscle would adversely affect these motions. The rectus femoris extends the hip. The vastus intermedius extends the knee. The semimembranosus extends the hip and flexes and medially rotates the knee. The sartorius assists in flexion and lateral rotation of the hip, as well as in medial rotation of the knee.

A 20-year-old man visits the family physician complaining of difficulty to flex and medially rotate his thigh while running and climbing. Which of the following muscles is most likely damaged in this individual? A. Rectus femoris B. Tensor fasciae latae C. Vastus intermedius D. Semimembranosus E. Sartorius

B. The calcaneofibular ligament is a round cord that passes posteroinferiorly from the tip of the lateral malleolus to the lateral surface of the calcaneus. A forced inversion of the foot can result in tearing of the calcaneofibular ligament and sometimes the anterior talofibular ligament as well. Both of these ligaments act to stabilize the foot and prevent an inversion injury. The plantar calcaneonavicular ligament supports the head of the talus. The long plantar ligament passes from the planter surface of the calcaneus to the groove on the cuboid and is important in maintaining the longitudinal arch of the foot. The short plantar ligament is located deep (superior) to the long plantar ligament and extends from the calcaneus to the cuboid and is also involved in maintaining the longitudinal arch of the foot. The deltoid (medial ligament of the ankle) attaches proximally to the medial malleolus and fans out to reinforce the joint capsule of the ankle.

A 22-year-old football player is admitted to the hospital with pain and swelling over the lateral aspect of the ankle. The emergency department doctor diagnoses an inversion sprain. Which ligament was most likely injured? A. Plantar calcaneonavicular (spring) B. Calcaneofibular C. Long plantar D. Short plantar E. Deltoid

A. The common fibular (peroneal) nerve passes around the head of the fibula and gives off deep (L4-5) and superficial fibular (peroneal) nerve (L5, S1-2) branches. The two nerves supply the dorsiflexors and evertors of the foot, respectively. In this case, the tibialis anterior and extensor digitorum longus are the only muscles listed that are supplied by either of these nerve branches, and both are innervated by the deep fibular (peroneal) nerve. The fibularis (peroneus) brevis and longus are innervated by the superficial fibular (peroneal) nerve and are evertors of the foot. The tibial nerve supplies each of the other muscles listed.

A 22-year-old male martial arts competitor was examined by the clinician because of pain and serious disability suffered from a kick to the side of his knee. Physical examination revealed a dark bruise just distal to the head of the fibula. Which of the following muscles will most likely be paralyzed? A. Tibialis anterior and extensor digitorum longus B. Tibialis posterior C. Soleus and gastrocnemius D. Plantaris and popliteus E. Flexor digitorum longus and flexor hallucis longus

D. The anterior cruciate ligament is attached to the anterior intercondylar area and the posterior part of the medial surface of the lateral femoral condyle. Posterior displacement of the femur on the tibia is prevented by the ACL. With the knee joint flexed, the ACL prevents the tibia from being pulled anteriorly. The posterior cruciate ligament is stronger, shorter, and broader, less oblique, and prevents anterior displacement of the femur on the tibia. Lateral collateral ligaments are cordlike and are attached proximally to the lateral side of the head of the fibula overlapped by the tendon of biceps femoris. The medial collateral ligament is a flat band and is attached above to the medial condyle of the femur and below to the medial surface of the shaft of the tibia. It is firmly attached to the edge of the medial meniscus and consequently is more prone to be injured. The oblique popliteal ligament is a tendinous expansion derived from the semimembranosus muscle. It strengthens the posterior aspect of the knee joint capsule. The patellar ligament (tendon) connects the lower border of the patella with the smooth convexity on the tuberosity of the tibia. It represents the continuation of the quadriceps tendon.

A 22-year-old male professional football player is admitted to the emergency department with acute right knee pain after sustaining a kick injury to an extended leg. A radiograph and a subsequent MRI revealed that the trauma caused anterior displacement of the tibia with respect to her femur. Which of the following ligaments was most likely injured? A. Fibular (lateral) collateral B. Tibial (medial) collateral C. Patellar D. Anterior cruciate E. Posterior cruciate F. Oblique popliteal

B. The tibial nerve divides into the medial and lateral plantar nerves on the medial side of the ankle. These two nerves provide sensation for the sole of the foot. Sensory supply to the dorsum of the foot is provided mostly by the superficial fibular (peroneal) nerve, with the deep fibular (peroneal) nerve providing sensation for the skin between the first and second toes. Foot drop would be caused by interruption of the common fibular (peroneal) nerve. Sensory loss to the lateral side of the foot results from loss of the sural nerve. Paralysis of the extensor digitorum brevis would be attributed to injury to the terminal motor branch of the deep fibular (peroneal) nerve.

A 22-year-old man is admitted to the emergency department after falling from his bicycle. Radiologic examination reveals a fracture of the tibia above the ankle. MRI and physical examination reveal that the tibial nerve is severed on the posterior aspect of the tibia. Which of the following signs will most likely be present during physical examination? A. Sensory loss of the dorsum of the foot B. Sensory loss on the sole of the foot C. Foot drop D. Paralysis of the extensor digitorum brevis E. Sensory loss of the entire foot

E. The posterior cruciate ligament runs from the posterior aspect of the intercondylar area of the tibia to the medial wall of the intercondylar fossa. It prevents posterior displacement of the tibia relative to the femur. This is usually tested with the posterior drawer test, in which the physician pushes the tibia backward while the knee is flexed in an attempt to displace it posteriorly. This is called the positive posterior drawer sign. The anterior cruciate ligament prevents anterior displacement of the tibia on the femur. The medial and lateral collateral ligaments stabilize the medial and lateral sides of the knee joint, respectively. The medial meniscus is an intracapsular fibrocartilage that improves the articulation of the femur and tibia.

A 22-year-old soccer player collides with one of her teammates. During examination on the field, the posterior drawer test was performed and the tibia moved backward in relation to her femur. Injury to which structure is confirmed by performing this test? A. Anterior cruciate ligament B. Lateral collateral ligament C. Medial collateral ligament D. Medial meniscus E. Posterior cruciate ligament

D. The gluteus maximus inserts into the gluteal tuberosity and the iliotibial tract. Although the gluteus maximus would continue to contract at the regions of insertion, their orientation would be displaced by the fracture. The gluteus medius, gluteus minimus, obturator internus, and piriformis all insert on some aspect of the greater trochanter of the femur

A 22-year-old woman is admitted to the emergency department after another vehicle collided with the passenger side of the convertible in which she was riding. Radiologic examination reveals an avulsion fracture of the greater trochanter. Which of the following muscles would continue to function normally if such an injury was incurred? A. Piriformis B. Obturator internus C. Gluteus medius D. Gluteus maximus E. Gluteus minimus

A. Gluteal injections should be given anterior and superior to a line drawn between the posterior superior iliac spine and the greater trochanter to avoid the sciatic nerve and other important nerves and vessels. Occasionally, one can encounter the lateral cutaneous branch of the iliohypogastric nerve, but this usually causes no serious problem. Certainly, one must stay anterior to a vertical line dropped from the highest point of the ilium. If the injected material is too near the sciatic nerve or other motor nerves, it can infiltrate the connective tissue sheath of the nerve, following the nerve, and result in major insult to the neural elements. The needle can cause trauma to this, or other nerves, likewise. Precautions to avoid the sciatic nerve are especially important in injecting the gluteal area in babies. The reduced dimensions are less "forgiving" in babies.

A 22-year-old woman is admitted with high fever and vaginal discharge. Physical and laboratory examinations reveal gonorrheal infection. A series of intramuscular antibiotic injections are ordered. Into which of the following parts of the gluteal region should the antibiotic be injected to avoid nerve injury? A. Anterior and superior to a line between the posterior superior iliac spine and the greater trochanter B. In the middle of a line between the anterior superior iliac spine and the ischial tuberosity C. Inferolateral to a line between the posterior superior iliac spine and the greater trochanter D. Inferomedial to a line between the posterior superior iliac spine and the greater trochanter E. Halfway between the iliac tuberosity and the greater trochanter

C. Incapacitation and unconsciousness from use of cocaine and other powerful narcotics have led to numerous cases of the "gluteal crush syndrome." Compression of the gluteal region while supine for extended periods of time can lead to gluteal crush injury, in which the nerves and vessels of the gluteal area are compressed. This can result in loss of gluteal muscles and other soft tissues and sciatic nerve compression. The nerve compression can cause paralysis of knee flexors and muscles of the anterior and lateral compartments of the leg, with sensory loss in the posterior thigh and leg and sensory loss in the foot. Tibial nerve loss would not result in loss of dorsiflexion of the foot nor generalized sensory loss. Neither piriformis entrapment nor femoral nerve entrapment is associated with loss of gluteal musculature, nor loss of knee flexion or plantar flexion of the foot, nor do they lead to general sensory loss in the limb.

A 22-year-old woman is found in a comatose condition, having lain for an unknown length of time on the tile floor of the courtyard. She is found in possession of cocaine. The patient is transported to the hospital while EMT personnel receive instructions for treatment of drug overdose. During the physical examination the patient's gluteal region shows signs of ischemia. After regaining consciousness, she exhibits paralysis of knee flexion and dorsal and plantar flexion and sensory loss in the limb. What is the most likely diagnosis? A. Tibial nerve loss B. S1-2 nerve compression C. Gluteal crush injury D. Piriformis entrapment syndrome E. Femoral nerve entrapment

E. One important function of the deltoid ligament is the prevention of excessive extension of the ankle. The ligament is so strong that excessive eversion can cause the medial malleolus to be pulled off (an avulsion fracture) rather than tearing the deltoid ligament.

A 23-year-old male basketball player injured his foot during training and is admitted to the emergency department. An MRI examination reveals a hematoma around the medial malleolus. Upon physical examination the patient shows excessive eversion of his foot. Which of the following ligaments most likely has a tear? A. Plantar calcaneonavicular (spring) B. Calcaneofibular C. Long plantar D. Short plantar E. Deltoid

D. Both the medial and lateral menisci are subject to rotational injuries and may be torn. The medial meniscus is much more liable to injury because it is attached to the fused deep layer of the medial collateral ligament and joint capsule. The lateral meniscus is separated from the fibular collateral ligament and is external to the capsule of the knee joint. Commonly seen in football players' knees, meniscal tears are usually diagnosed by MRI or by arthroscopy. The presenting symptoms of tearing may be pain and swelling, or locking of the knee. Locking of the knee suggests a bucket handle tear, in which a partly detached cartilage wedges between the tibia and femur, inhibiting further movement. A bucket handle tear is often associated with rupture of the anterior cruciate ligament. Sometimes a momentary click can be heard in flexion/extension movements of the knee. Meniscectomy is a successful operation, but currently there is greater emphasis on repairing small tears. Meniscal cysts can form secondary to meniscal tears and some of these can also be treated arthroscopically.

A 23-year-old man is admitted to the emergency department after injuring his knee while playing football. During physical examination there is pain and swelling of the knee, in addition to locking of the knee in full extension. Radiologic examination reveals a bucket handle meniscal tear (Fig. 5-12). Which of the following ligaments is most likely injured? A. Posterior cruciate B. Medial collateral C. Lateral collateral D. Anterior cruciate E. Coronary

A. The ventral ramus of L4 contains both sensory and motor nerve fibers. Injury from a stab wound could result in loss of sensation from the dermatome supplied by this segment. A dermatome is an area of skin supplied by a single spinal nerve; L4 dermatome supplies the medial aspect of the leg and foot. Loss of the Achilles tendon reflex relates primarily to an S1 deficit. The Achilles tendon reflex is elicited by tapping the calcaneus tendon, which results in plantar flexion. The obturator internus and gluteus medius and minimus are responsible for abduction of the thigh and are innervated by nerves L4, L5, and S1 (with L5 usually dominant). Nerves L5, S1, and S2 are responsible for eversion of the foot (S1 dominant)

A 23-year-old man is admitted to the emergency department with a deep, bleeding stab wound of the pelvis. After the bleeding has been arrested, a magnetic resonance imaging (MRI) examination gives evidence that the right ventral primary ramus of L4 has been transected. Which of the following problems will most likely be seen during physical examination? A. Reduction or loss of sensation from the medial aspect of the leg B. Loss of the Achilles tendon reflex C. Weakness of abduction of the thigh at the hip joint D. Inability to evert the foot E. Reduction or loss of sensation from the medial aspect of the leg and loss of Achilles tendon reflex

B. The original axial vessel of the lower limb is retained as the (usually tiny) sciatic branch of the inferior gluteal artery. In some cases this vessel is retained as the primary proximal vessel to the limb, wherein there is hypoplastic development of the femoral artery. Aneurysms of the enlarged sciatic artery in the gluteal region are relatively common, as is rupture of the vessel (with profuse bleeding) if they are exposed in the gluteal area. The profunda femoris or deep femoral branch of the femoral artery usually provides three perforating branches to the posterior compartment, but not a branch such as that described. The descending branch of the medial circumflex femoral anastomoses with the first perforator. The superior gluteal artery anastomoses with the inferior gluteal by a descending branch or branches. The descending branch of the lateral circumflex femoral is the descending genicular artery, which anastomoses with the superior lateral genicular branch of the popliteal artery

A 23-year-old man is admitted to the emergency department with pain and cyanosis of his right lower limb. Doppler ultrasound studies reveal deficiency in development of his femoral artery, which appears to terminate mid thigh. A thrombotic occlusion is seen in an unusual, rather tortuous, large vessel in the posterior compartment of the thigh, arising in the gluteal area and continuous inferiorly with a normal appearing popliteal artery. It is decided that a vascular graft should be placed from the femoral artery to the popliteal artery. What is the identity of the aberrant artery? A. A large, fifth perforating branch of the femoral B. An sciatic branch of the inferior gluteal artery C. Descending branch of the medial circumflex femoral D. Descending branch of the superior gluteal artery E. An enlarged descending lateral circumflex femoral artery

B. The psoas muscle arises from the base of the transverse processes, the sides of the vertebral bodies, and the intervertebral discs, from the twelfth thoracic to the fifth lumbar vertebrae and inserted into the lesser trochanter of the femur. The psoas flexes the thigh at the hip joint on the trunk, or if the thigh is fixed, it flexes the trunk on the thigh, as in sitting up from a lying position. The inflamed appendix is pushed up against the peritoneum from the the contracted psoas. As a result it is in touch with the parietal peritoneum, producing acute pain. In some other cases it may retain the purulence of a psoas abscess, and spinal tuberculosis may present as a cold abscess in the groin. The psoas is enclosed in a fibrous sheath that is derived from the lumbar fascia. The sheath is not part of the lumbar fascia but the lateral edge blends with the anterior layer of that fascia.

A 23-year-old man is taken to the emergency department because of anorexia, nausea, vomiting, and severe abdominal pain in the right lower quadrant. On examination, he has tenderness in the right lower quadrant with rebound tenderness. The physician suspects appendicitis. To confirm this diagnosis, the physician attempts to straighten the patient's flexed thigh. This causes the patient to wince with pain. Which of the following muscles most likely caused this symptom? A. Adductor magnus B. Psoas major C. Biceps femoris D. Obturator internus E. Gluteus maximus

C. Mesenchyme between digital rays undergoes apoptosis for the digits to form. Failure or incomplete apoptosis usually results in fused digits (syndactyly). This may involve the skin and soft tissues alone or may include the bone. Digital rays form from the hand plate. Failure of development of any digital ray results in underdevelopment of a finger or toe. Neural crest cells do not contribute to the formation of the foot. The zone of polarizing activity modulates the patterning of the limb in the anteroposterior diameter. The abnormality described did not involve the phalanges, as shown by radiograph, and thus could not have been caused by faulty chondrification.

A 23-year-old woman delivered a live male infant at 37 weeks' gestation after an uneventful pregnancy. Examination of the infant revealed the right second and third toes were fused. Radiographs of the right foot indicated 14 phalanges in their correct position. After review by the pediatric orthopedic surgeon, corrective surgery for the deformity was scheduled. Which of the following embryologic conditions explains the infant's condition? A. The digital ray for the third toe did not develop B. Excessive neural crest cell migration into the foot C. Incomplete apoptosis of tissue between digital rays D. Lack of signal from the zone of polarizing activity (ZPA) E. Faulty development of chondrification centers

A. The skin of the anterior medial thigh and medial leg and foot is supplied by the femoral nerve. The saphenous nerve is a branch of the femoral and only supplies the medial leg and foot. The obturator supplies the medial and medial posterior aspect of the thigh. The tibial nerve supplies the skin of the posterolateral leg, lateral ankle and foot and sole of the foot. The fibular (peroneal) nerve supplies the skin over the lateral aspect of the leg and dorsal aspect of the foot.

A 23-year-old woman was taken to the emergency department after being involved in a head-on collision with a truck. On physical examination a hematoma was seen in the medial thigh. A CT scan revealed a fracture of the femur with a ruptured femoral artery. She was taken to the operating room for repair of the damaged structures. Two days postoperatively during physical examination the patient has loss of sensation to the anterior medial thigh and medial side of her leg and foot. Branches of which of the following nerves were most likely injured in the repair of the fracture? A. Femoral B. Saphenous C. Obturator D. Tibial E. Fibular (peroneal)

D: The saphenous nerve is a cutaneous branch of the femoral nerve and supplies the skin on the medial leg along the great saphenous vein. Middle clunial is a posterior division of S1 to S3. Lateral femoral cutaneous is from L2 to L3 and the lumbar plexus but is not a femoral nerve branch. Superficial fibular (peroneal) nerve is a branch from the common fibular (peroneal) nerve.

A 24-year-old female motocross racer was involved in a crash that left her right leg pinned under her bike. After the accident, she could no longer extend her right knee. The affected nerve gives rise to which of the following cutaneous nerves? A. Middle clunial nerve B. Lateral femoral cutaneous branch C. Superficial fibular (peroneal) nerve D. Saphenous nerve E. Deep fibular (peroneal)

B: Extensors of the knee are the quadriceps femoris muscle, which attaches to the tibia via the patellar ligament. This muscle group is supplied by the femoral nerve. The sciatic nerve supplies the posterior thigh muscles not the anterior compartment. The obturator supplies the medial compartment and the saphenous nerve does not supply motor innervation to any muscles but is a cutaneous branch of the femoral nerve

A 24-year-old female motocross racer was involved in a crash that left her right leg pinned under her bike. After the accident, she could no longer extend her right knee. Which of the following nerves was most likely affected? A. Sciatic nerve B. Femoral nerve C. Obturator nerve D. Saphenous nerve E. Tibial nerve

B. The gastrocnemius muscle arises from the femur just proximal to the femoral condyles. This strong muscle could displace the distal fragment of the fractured femur posteriorly. In addition, the popliteal artery is the deepest structure in the popliteal fossa (right against the popliteal surface of the distal femur) and is susceptible to laceration in this scenario as the fractured end of the distal femoral fragment is pulled against the popliteal artery. Orthopedic surgeons always look for damage to the popliteal artery in a patient with a supracondylar fracture. The soleus arises from the tibia and would have no effect upon the femur. The semitendinosus arises from the ischial tuberosity and inserts medially on the proximal tibia, via the pes anserinus. The tibialis anterior arises from the tibia and inserts mostly onto the navicular bone.

A 24-year-old man is admitted to the emergency department after a car collision. Radiologic examination reveals a fracture at the junction of the middle and lower thirds of the femur. An MRI examination provides evidence that the popliteal vessels were injured when the distal fragment of the fracture was pulled posteriorly. Which of the following muscles is most likely to displace the distal fracture fragment? A. Soleus B. Gastrocnemius C. Semitendinosus D. Gracilis E. Tibialis anterior

C. A severe injury of the tibial nerve in the popliteal fossa would result in a dorsiflexed and everted foot because of the intact muscles of the extensor (anterior) and evertor (lateral) compartments of the leg. It would result also in some weakening of knee flexion because of loss of the gastrocnemius muscle, which flexes the knee and plantar flexes the foot. The hamstrings also flex the knee, so this function would not be lost. Plantar flexion at the ankle would be paralyzed with the loss of the gastrocnemius and soleus, in addition to the flexors of the toes, and inversion by the tibialis posterior. Foot drop results from loss of the anterior compartment, innervated by the deep fibular (peroneal) nerve.

A 24-year-old woman received a small-caliber bullet wound to the popliteal fossa from a drive-by assailant. The patient was admitted to the emergency department, where the surgeons recognized that the bullet had severed the tibial nerve. Such an injury would most likely result in which of the following? A. Inability to extend the leg at the knee B. Foot drop C. A dorsiflexed and everted foot D. A plantar flexed and inverted foot E. Total inability to flex the leg at the knee joint

A. The femoral vein lies medial to the femoral artery in the femoral sheath. The femoral sheath is broken into three compartments: lateral, intermediate, and medial. The lateral compartment contains the femoral nerve. The medial compartment encloses the femoral canal and consists of lymphatic tissue and a lymph node, plus areolar tissue. The intermediate contains the femoral vein.

A 25-year-old man, an intravenous drug abuser, had been injecting himself with temazepam (a powerful intermediate acting drug in the same group as diazepam (Valium) and heroin for 5 years, leaving much residual scar tissue over points of vascular access. The patient is admitted to the emergency department for a detoxification program requiring an intravenous infusion. The femoral veins in his groin are the only accessible and patent veins for intravenous use. Which of the following landmarks is the most reliable to identify the femoral veins? A. The femoral vein lies medial to the femoral artery. B. The femoral vein lies within the femoral canal. C. The femoral vein lies lateral to the femoral artery. D. The femoral vein lies directly medial to the femoral nerve. E. The femoral vein lies lateral to the femoral nerve.

C. In an inversion injury the most common ligament involvement comes from the anterior talofibular and calcaneofibular ligaments. The medial plantar nerve is medially located within the sole of the foot and might be injured by traction in an eversion injury, not an inversion injury. The posterior talofibular ligament is located posteriorly and is not usually injured in an inversion injury. The deltoid ligament is located medially and would be injured with an eversion injury; it is so strong, however, that eversion is more likely to fracture the medial malleolus rather than tear the deltoid ligament.

A 27-year-old female tennis pro injured her ankle during the quarterfinal match. A physical examination at the outpatient clinic revealed a severe inversion sprain of the ankle. Which of the following structures is most commonly damaged in such injuries? A. Medial plantar nerve B. Tibial nerve C. Anterior talofibular ligament D. Posterior talofibular ligament E. Deltoid ligament

B. The tendons of the semitendinosus and semimembranosus provide the superior medial border of the popliteal fossa. The semitendinosus inserts with the pes anserinus on the proximal, medial tibia. The semimembranosus inserts on the tibia posteriorly. The biceps femoris forms the superior lateral border of the fossa, as the tendon passes to insertion on the fibula. The plantaris arises from the femur just above the lateral head of the gastrocnemius, passing distally to insert on the calcaneus via the tendo Achilles. The popliteus arises from the tibia and passes superiorly and laterally to insert on the lateral condyle of the femur, with a connection to the lateral meniscus.

A 27-year-old male triathlon competitor complained that he frequently experienced deep pains in one calf that almost caused him to drop out of a regional track-and-field event. Doppler ultrasound studies indicated, and surgical exposure confirmed, the existence of an accessory portion of the medial head of the gastrocnemius that was constricting the popliteal artery. Above the medial head of the gastrocnemius, the superior medial border of the popliteal fossa could be seen. Which of the following structures forms this border? A. Tendon of biceps femoris B. Tendons of semitendinosus and semimembranosus C. Tendon of plantaris D. Adductor hiatus E. Popliteus

D. The deep fibular (peroneal) nerve is responsible for sensation over the first web space of the foot. Dorsiflexion and inversion of the ankle is produced by the muscles supplied by the deep fibular (peroneal) nerve. The nerves responsible for the knee jerk reflex, knee flexion, eversion, and plantar flexion are all located superior to the location of the tumor and will not be damaged. The nerve located in the space between the tibia and fibula is the deep fibular (peroneal) nerve.

A 27-year-old man has had increasing difficulty walking and complained of an area of numbness on the dorsum of his right foot. Examination reveals a hard mass at the anterolateral aspect of his right leg just below the knee. Imaging studies reveal a large bone tumor between the fibula and tibia that is compressing a nerve, accounting for his neurological symptoms. Which of the following is the most likely description of abnormalities on neurological examination? A. Decreased/absent knee jerk reflex and decreased sensation on the medial aspect of the leg B. Weakness of flexion at the knee and decreased sensation of the plantar aspect of the foot C. Weakness of eversion at the ankle and decreased sensation between the first and second toes D. Weakness of inversion, dorsiflexion at the ankle, and decreased sensation between the first and second toes E. Weakness of plantar flexion at the ankle, weakness of toe flexion, decreased sensation of the plantar aspect of the foot

D. The lesser (short) saphenous vein ascends up the middle of the calf from beneath the lateral malleolus, most commonly terminating at the popliteal fossa by piercing the deep fascia and joining the popliteal vein. The popliteal vein is the most superficial of major structures deep to the deep popliteal fascia. The perforating tributaries of the deep femoral vein drain to the deep femoral vein of the posterior compartment of the thigh, thereafter into the femoral vein. The superior medial genicular vein is a tributary to the popliteal vein.

A 27-year-old woman had suffered a penetrating injury in the popliteal region by an object thrown from a riding lawnmower. She was admitted to the emergency department for removal of the foreign object. After making a midline incision in the skin of the popliteal fossa, the surgical resident observed a vein of moderate size in the superficial tissues. What vein would be expected at this location? A. Popliteal vein B. Perforating tributary to the deep femoral vein C. Great saphenous vein D. Lesser (short) saphenous vein E. Superior medial genicular vein

D. The inferior gluteal nerve supplies the gluteus maximus muscle, which extends and laterally rotates the hip joint; through the iliotibial tract, it also extends the knee joint. The superior gluteal nerve supplies the gluteus medius and minimus and tensor fasciae latae muscles, which work together as medial rotators of lower limb. The nerve to the piriformis supplies the piriformis muscle, which laterally rotates the femur with hip extension and abducts the femur with hip flexion.

A 29-year-old construction worker falls onto some rusty wire mesh and suffers a deep laceration to his right buttock. When the ambulance arrives to transport him to the emergency department and it is noted that he has difficulty stepping up into the ambulance with his right leg. Which nerve has probably been damaged? A. Superior gluteal B. Tibial C. Common fibular (peroneal) D. Inferior gluteal E. Nerve to piriformis

E. The lateral femoral cutaneous nerve leaves the pelvis laterally, about 2 cm medial to the anterior superior iliac spine, passing beneath, or through, the inguinal ligament. As a consequence of its site of exit, any tension upon or compression of the inguinal ligament can affect the nerve. If it is thus affected, the individual may feel burning sensations or pain along the lateral aspect of the thigh, which is the region of distribution of the nerve. Obesity, sudden weight loss, wearing a heavy gun belt, wearing trousers that are too tight (Calvin Klein syndrome), or having someone sitting on another's lap for an extended period of time can lead to meralgia paresthetica, the painful lateral thigh. The femoral nerve emerges from beneath the middle of the inguinal ligament and is not usually affected by similar traction or compression. The obturator nerve leaves the pelvis through the obturator canal and enters the thigh deeply in a protected location. It innervates the adductor muscles and supplies sensation on the medial aspect of the thigh. The fibular (peroneal) division of the sciatic nerve supplies the muscles of the anterior and lateral compartments of the leg and provides sensory fibers for the dorsum and lateral side of the foot. The superior gluteal nerve provides motor supply to the gluteus medius and minimus muscles.

A 29-year-old male police officer is examined in a neighborhood clinic, with a complaint of discomfort in the lateral thigh. The physician observes that the policeman is rather overweight and that he is wearing a heavy leather belt, to which numerous objects are attached, including his empty holster. After a thorough physical examination a diagnosis of meralgia paresthetica is confirmed. Which of the following nerves is most likely involved? A. Superior gluteal B. Femoral C. Obturator D. Fibular (peroneal) division of sciatic E. Lateral femoral cutaneous

A. The common fibular (peroneal) nerve is a branch of the sciatic nerve. It descends on the lateral side of the popliteal fossa before winding around the head of the fibula. It then divides into superficial and deep nerves that supply the lateral and anterior compartments of the leg respectively. Due to its superficial course, it is easily injured in patients with long leg casts (which run from just below the knee). The nerve supplies the dorsiflexors of the leg, the skin of the first web space (via the deep fibular), the evertors of the foot, and the skin of the lateral side of the leg and dorsum of the foot (via the superficial fibular).

A 29-year-old man is brought to the physician for removal of a cast from his left leg. He had sustained a fracture of the left lower extremity 6 weeks prior which was immobilized in a cast that extended from just below the knee to the foot. At the time of injury, there was severe pain but normal strength in the extremity. When the cast was removed, physical examination showed a pronounced left foot drop with paresthesia and sensory loss over the dorsum of the left foot and lateral leg. Injury to which nerve is the most likely cause? A. Common fibular (peroneal) B. Superficial fibular (peroneal) C. Deep fibular (peroneal) D. Sciatic E. Tibial

A. The obturator nerve is a branch of the lumbar plexus that originates from L2 to L4. It descends medial to the psoas on the posterior abdominal wall into the pelvis where it runs along the lateral wall of the lesser pelvis, above and anterior to the obturator vessels. It enters into the medial thigh via the obturator canal (an opening above the obturator membrane) to supply the obturator externus muscle and the adductors of the thigh. The femoral nerve innervates the anterior compartment of the thigh. The inferior gluteal innervates the gluteus maximus muscle, while the superior gluteal innervates the gluteus minimus and medius. The tibial nerve innervates the posterior compartment of the lower limb.

A 29-year-old woman is involved in a car crash and is taken to the emergency department. Radiographs reveal a fracture of her pelvis. During healing of the pelvic fracture, a nerve becomes entrapped in the bone callus. Musculoskeletal examination reveals an inability to adduct the thigh. Which of the following nerves is most likely affected? A. Obturator B. Femoral C. Inferior gluteal D. Superior gluteal E. Tibial

B. Injury to the superior gluteal nerve results in a characteristic motor loss, with paralysis of the gluteus medius and minimus. In addition to their role in abducting the thigh, the gluteus medius and minimus function to stabilize the pelvis. When the patient is asked to stand on the limb of the injured side, the pelvis descends on the opposite side, indicating a positive Trendelenburg test. The gluteal, or lurching, gait that results from this injury is characterized by the pelvis drooping to the unaffected side when the opposite leg is raised. In stepping forward, the affected individual leans over the injured side when lifting the good limb off the ground. The uninjured limb is then swung forward. The gluteus maximus, supplied by the inferior gluteal nerve, is the main muscle responsible for allowing a person to rise to a standing position (extending the flexed hip). Spinal nerve roots L1 and L2 and the femoral nerve are responsible for hip flexion. Injury to the left superior gluteal nerve would result in sagging of the right side of the pelvis when the affected individual stands on the left limb. The hamstring muscles, mainly responsible for flexing the knees to allow a person to sit down from a standing position, are innervated by the tibial branch of the sciatic nerve.

A 30-year-old man suffered a superior gluteal nerve injury in a motorcycle crash in which his right lower limb was caught beneath the bike. He is stabilized in the emergency department. Later he is examined and he exhibits a waddling gait and a positive Trendelenburg sign. Which of the following would be the most likely physical finding in this patient? A. Difficulty in standing from a sitting position B. The left side of the pelvis droops or sags when he attempts to stand with his weight supported just by the right lower limb C. The right side of the pelvis droops or sags when he attempts to stand with his weight supported just by the left lower limb D. Weakened flexion of the right hip E. Difficulty in sitting from a standing position

B. The suprapatellar bursa is found in the anterior surface of the inferior part of the femur under the quadriceps femoris muscle. A high-riding superior patellar fragment lead to the exposure of this bursa. In traumatic episodes following this condition, blood and fat from the knee can enter into the suprapatellar bursa. The popliteal bursa is located behind the knee, away from the site of injury. The superficial infrapatellar bursa is located below the patella, between the patella ligament and the skin. The deep infrapatellar bursa is located below the site of the injury, between the upper part of the tibia and the patella ligament. The gastrocnemius bursa is at the back of the knee, as well as below the level of the injury.

A 30-year-old man was brought to the emergency department after his involvement in a motor vehicle crash. He complained of pain in the right knee and an inability to bear weight. On examination, there were minor abrasions about his body, in addition to a deep, 5 cm oblique laceration over the anterior right knee, which exposed the patella. He was unable to extend the right knee. Radiographs revealed a displaced transverse fracture of the inferior pole of the patella. The superior fragment of the patella appeared to be "high riding" over the anterior surface of the femur. Which of the following most likely occurred? A. Blood and fat from the injury can enter the popliteus bursa B. Blood and fat from the injury can enter the suprapatellar bursa C. Joint fluid can enter the subcutaneous infrapatellar bursa D. The deep infrapatellar bursa will be affected E. The gastrocnemius bursa will not be affected

E. The obturator membrane is a fibrous sheet that almost completely closes the obturator foramen, leaving a small gap, the obturator canal, for the passage of the obturator nerve and vessels as they leave the pelvis to enter the medial thigh. The femoral canal is the small medial compartment for the lymph vessels. It is about 0.5 in (1.3 cm) long, and its upper opening is called the femoral ring. It has following borders: anteriorly the inguinal ligament; posteriorly the superior ramus of the pubis; medially the lacunar ligament; and laterally the femoral vein. A triangular shaped defect in the external oblique aponeurosis lies immediately above and medial to the pubic tubercle. This is known as the superficial inguinal ring. The deep ring is an oval opening in the transversalis fascia and lies about 0.5 in (1.3 cm) above the inguinal ligament, midway between the anterior superior iliac spine and the pubic symphysis. Fossa ovalis, which refers to an oval opening in the superomedial part of the fascia lata of the thigh, lies 3 to 4 cm inferolateral to the pubic tubercle.

A 30-year-old man who is a bodybuilder presents to the physician's office complaining of pain and tingling sensation radiating down the inside of his thigh that was exacerbated upon thigh movement. A hernia through which opening would most likely cause this presentation? A. Femoral ring B. Superficial inguinal ring C. Deep inguinal ring D. Fossa ovalis E. Obturator canal

E. The muscles of the limbs develop from the myotome component of the somites. The somites are condensations of paraxial mesoderm that form after the formation of the trilaminar disc. The lateral plate mesoderm gives rise to the serous membranes, coverings of organs and the heart. The intermediate mesoderm gives rise to the urogenital system and its accessory glands. The chondrification centers precede the formation of the muscles, as it gives the skeletal framework. Neural crest cells give rise to dorsal root ganglia, leptomeninges, Schwann cells, sympathetic ganglia, and the chromaffin cells of the adrenal medulla.

A 30-year-old woman is admitted to the emergency department with complaints of pain to the anterior left thigh. While participating in a 100-meter race, she felt a sudden onset of pain in the anterior mid thigh area and could only limp to the finish line. Physical examination revealed a swollen, tender right thigh anteriorly. Extension of the knee was limited due to pain. Ultrasonography of the area revealed a defect in the fibers of the quadriceps muscle, confirmed by CT scan of the limb. Which of the following is the embryologic origin of the affected structure? A. Lateral plate mesoderm B. Dorsolateral migration of neural crest cells C. Preceded the development of chondrification centers D. Intermediate mesoderm E. Migration of cells from paraxial mesoderm

E. The paraxial mesoderm develops into somites. Limb muscles develop from the ventral myotome of the somites in response to molecular signals. Embryological derivatives of the lateral plate mesoderm include the circulatory and gut wall, body wall lining, and dermis. Derivative of the neural crest cells does not include the limb muscles. Chondrification is associated with cartilage formation and not muscles. The intermediate mesoderm eventually thins out laterally and becomes the mesoderm, which gives the circulatory and gut walls, plus the lining of the body wall and dermis.

A 30-year-old woman is admitted to the emergency department with complaints of pain to the anterior left thigh. While participating in a 100-meter race, she felt a sudden onset of pain in the anterior mid thigh area and could only limp to the finish line. Physical examination revealed a swollen, tender right thigh anteriorly. Extension of the knee was limited due to pain. Ultrasonography of the area revealed a defect in the fibers of the quadriceps muscle, confirmed by CT scan of the limb. Which of the following is the embryologic origin of the affected structure? A. Lateral plate mesoderm B. Dorsolateral migration of neural crest cells C. Preceded the development of chondrification centers D. Intermediate mesoderm E. Migration of cells from paraxial mesoderm

B. The obturator membrane is a thin membrane that covers the obturator foramen except at its superior part. The obturator nerve exits the pelvis and enters into the medial compartment of the thigh by passing through the obturator canal alongside the obturator vessels. Traumatic injuries to the membrane will most likely lead to obturator nerve damage. The obturator nerve supplies motor innervations to the adductor muscles of the thigh (gracilis, obturator externus, adductor longus, adductor brevis and a portion of the adductor magnus). It also provides sensory innervation to the medial aspect of the thigh. Urinary and fecal incontinence is mediated by autonomic nerves and the pudendal nerve. Both nerves have no relationship with the obturator membrane. The gluteus medius and minimus muscles are the main hip abductors. They also stabilize the hip on the swing-side during motion. These muscles are supplied by the superior gluteal nerve, which leaves the pelvis through the greater sciatic foramen above the piriformis muscle. Flexors of the hip found in the anterior compartment of the thigh are innervated by the femoral nerve, which has no relationship with the obturator membrane. The sciatic nerve supplies the muscles in the posterior compartment of the thigh and also sends cutaneous innervations to the skin of the posterior thigh. It enters the posterior compartment of the thigh from the gluteal region.

A 30-year-old woman was admitted to the emergency department after being involved in a motor vehicle crash. The patient complained of pain to the right hip and knee. During physical examination, there is no deformity of the lower limb, but there is tenderness over the right ischiopubic ramus. Pelvic radiographs revealed an inferiorly displaced fracture of the right superior and inferior pubic rami with dislocation of the right sacroiliac joint and pubic symphysis. The patient was referred to the orthopedics team, which had a high suspicion of rupture of the right obturator membrane. What clinical findings are most likely to be present in this patient? A. Urinary and fecal incontinence and diminished sensation over the perineum B. Weak adduction of the hip and diminished sensation over the upper medial thigh C. Weak abduction of the hip and positive Trendelenburg sign D. Weak flexion of the hip and diminished sensation over the anterior thigh and medial leg E. Weak extension of the hip and diminished sensation over the posterior thigh

D. The sural nerve is formed by contributions from the tibial nerve and a branch from the common fibular (peroneal) nerve. It provides sensation for the lower lateral portion of the calf and continues beneath the lateral malleolus as the lateral cutaneous nerve of the foot. It is often used for nerve grafting procedures as well as biopsied for diagnostic purposes. When it is grafted to the "living end" of a cut motor or sensory nerve, the severed nerve processes within the "living" nerve grow into the sural nerve sheath, using it as a guide to the distal, surgically anastomosed nerve. Thus, axons from a branch of a functional motor nerve can grow to reinnervate paralyzed muscles. In this case, the surgeon would connect portions of the sural nerve to the functional facial nerve, tunnel it to the opposite side of the face, and join it surgically to the branches of the paralyzed nerve, where it would grow through the now empty nerve sheaths (due to Wallerian degeneration) to the muscles. Growth and reinnervation usually occur at a rate of 1 mm/day (or 1 inch/month) so the time estimated before reinnervation is based on the distance the regenerating fibers need to traverse. The tibial nerve supplies muscles and sensation to the calf and plantar surface of the foot. The common fibular (peroneal) nerve innervates the lateral and anterior compartment muscles and sensation to the dorsum of the foot. The saphenous nerve accompanies the great saphenous vein on the medial side of the leg and foot.

A 31-year-old woman presents to the department of surgery with a complaint of facial paralysis (Bell's palsy), which had appeared a year earlier and had resulted in paralysis of muscles of one side of her face. The chief of plastic surgery recommends a nerve graft, taking a cutaneous nerve from the lower limb to replace the defective facial nerve. The surgery is successful. Six months after the procedure, there is restoration of function of previously paralyzed facial muscles. There is an area of skin on the back of the leg laterally and also on the lateral side of the foot that has no sensation. What nerve was used in the grafting procedure? A. Superficial fibular (peroneal) B. Tibial C. Common fibular (peroneal) D. Sural E. Saphenous

B. Pott's fracture is a rather archaic term for a fracture of the fibula at the ankle. The term is often used to indicate a bimalleolar fracture of fibula and tibia, perhaps with a tear in the medial collateral ligament, allowing the foot to be deviated laterally. (The medial malleolus will often break before the deltoid ligament tears.) This fracture is also known as Dupuytren's fracture. The fracture results from abduction and lateral rotation of the foot in extreme eversion. There can also be fracture of the posterior aspect of the distal tibia. The spring ligament, also known as the plantar calcaneonavicular ligament, extends from the calcaneus to the navicular bone and is a part of the medial longitudinal arch. This ligament would not be affected in eversion or inversion of the ankle. The plantar ligament, which is composed of the long and short plantar ligaments, supports the lateral longitudinal arch of the foot and would therefore not be affected by inversion or eversion of the foot. The calcaneofibular ligament runs from the calcaneus to the fibula. It would be injured during inversion of the foot, not in eversion, as is the case in a Pott's fracture

A 32-year-old male basketball player comes down hard on his ankle. He is admitted to the outpatient clinic, and radiologic examination reveals a Pott's fracture. What ligament is most likely injured? A. Calcaneofibular ligament B. Deltoid ligament C. Spring ligament D. Plantar ligament E. Long plantar ligament

C. If the needle is inserted about 1.5 cm lateral to the maximal femoral pulse, it will intersect the femoral nerve in most cases. (Fluoroscopic or ultrasound guidance is advisable to avoid iatrogenic errors.) The deep inguinal ring is located about 4 cm superolateral to the pubic tubercle and very close to the origin of the inferior epigastric vessels from the external iliac artery and vein. The approximate site of exit of the lateral femoral cutaneous nerve from the abdomen is 1.5 cm medial to the anterior superior iliac spine. Injections 1.5 cm medial to the femoral artery pulse will enter the femoral vein. Midway between the anterior superior iliac spine and the pubic symphysis can vary approximately 1.5 cm either medial or lateral from the femoral artery.

A 32-year-old man is admitted to the emergency department after a car collision. Radiologic examination reveals a distal fracture of the femur. The patient is in severe pain, and a femoral nerve block is administered. What landmark is accurate for localizing the nerve for injection of anesthetics? A. 1.5 cm superolateral to the pubic tubercle B. 1.5 cm medial to the anterior superior iliac spine C. 1.5 cm lateral to the femoral pulse D. 1.5 cm medial to the femoral pulse E. Midway between the anterior superior iliac spine and pubic symphysis

C. The bone to which the injured ligament attaches is the calcaneus. The navicular bone, located medially in the foot, articulates posteriorly with the head of the talus and anteriorly with the cuneiform bones. The cuboid bone of the lateral longitudinal arch articulates posteriorly with the calcaneus. The talus articulates with the tibia and fibula in the ankle joint mortise.

A 32-year-old man is admitted to the emergency department after an injury to his foot while playing football with his college friends. An MRI examination reveals multiple tendinous tears (Fig. 5-8). Which of the following bones is associated with the muscle tears? A. Navicular B. Cuboid C. Calcaneus D. Sustentaculum tali E. Talus

C. The deep fibular (peroneal) nerve is a branch of the common fibular (peroneal) nerve and begins at about the level of the neck of the fibula, between it and the fibularis (peroneus) longus. This nerve supplies the extensors of the foot (extensor digitorum longus, fibularis [peroneus] tertius, extensor hallucis longus, tibialis anterior, extensor digitorum brevis, and extensor hallucis brevis). It innervates the first web space of the foot. Fracture of the head of the fibula can damage this nerve, resulting in a high stepping gait and numbness over the dorsum and first web space of the foot. Muscles in the posterior compartment of the leg are involved in planter flexion. These muscles are innervated by the tibial nerve, which is a continuation of the sciatic nerve; its medial cutaneous branch supplies sensation to the posteromedial side of the leg. Waddling gait and numbness at the anterolateral side of the leg are associated with the superficial fibular and the lateral sural cutaneous nerves.

A 32-year-old man is brought to the emergency department with complaints of pain to the left ankle and knee. The patient recalls that during a football game, his left foot landed in a hole as he was running on an uneven dirt field. The ankle was externally rotated and everted while the knee twisted medially. He was unable to bear weight subsequently. During physical examination, the right ankle is swollen and there is exquisite tenderness over the right medial malleolus and the proximal lateral leg. Radiologic examination of the right lower limb reveals a displaced fracture of the neck of right fibula and a comminuted fracture of the tibial plafond and medial malleolus. Which of the following describes the most likely consequences of this injury? A. Weak "push-off" while walking and numbness over the posteromedial leg B. Weak ankle eversion and numbness over the dorsum of the foot C. High stepping gait and numbness over the dorsum and first web space of the foot D. Waddling gait and inability to feel a pin prick over the anterolateral leg E. Swing-out gait and numbness over the medial leg

C. Forceful external rotation and eversion of the ankle often leads to this type of injury as the bony components are pushed apart forcefully. It is commonly referred to as a Pott fracture (the medial malleolus is pulled forcefully by the strong deltoid ligament as the talus moves laterally, causing a fracture of the lateral malleolus). Inversion, extreme plantar flexion, and forceful dorsiflexion cause a rupture of the ligaments. Direct upward force of the talus is usually due to a fall from great height and will damage the spine and calcaneus.

A 32-year-old man was brought to the emergency with complaints of pain to the left ankle and leg. During a game of football, his left foot landed in a hole as he was running on an uneven dirt field. He was unable subsequently bear weight on his left limb. On examination, the right ankle was swollen, with exquisite tenderness over the right medial malleolus and over the distal third of the lateral right leg. Radiographs of the right lower limb revealed an inferiorly displaced fracture of the right medial malleolus and a spiral fracture of the distal third of the right fibula. Which of the following describes the most likely mechanism of this injury? A. Forceful inversion of the ankle B. Direct upward force from the talus into the tibial plafond C. Forceful external rotation and eversion of the ankle D. Forceful dorsiflexion of the foot E. Extreme plantar flexion of the foot

A. The gluteus maximus is innervated by the inferior gluteal nerve, and this muscle is responsible for extension and lateral rotation of the thigh. It is the primary muscle that extends the flexed hip and is used to rise from a seated position. The gluteus minimus is innervated by the superior gluteal nerve and is responsible for abduction of the thigh. Hamstring muscles are innervated by the tibial portion of the sciatic nerve, and these are responsible for extension of the thigh and flexion of the leg. The iliopsoas muscle is innervated by L1 and L2 and the femoral nerve, and flexes the thigh. The obturator internus is innervated by the nerve to the obturator internus and is a lateral rotator of the thigh.

A 32-year-old patient received a badly placed intramuscular injection to the posterior part of his gluteal region. The needle injured a motor nerve in the area. Later, he had great difficulty rising to a standing position from a seated position. Which muscle was most likely affected by the injury? A. Gluteus maximus B. Gluteus minimus C. Hamstrings D. Iliopsoas E. Obturator internus

D. Inflammation of the plantar aponeurosis is referred to as plantar fasciitis. Plantar fasciitis is a common clinical condition that results from tearing or inflammation of the tough band of tissue stretching from the calcaneus to the ball of the foot (the plantar aponeurosis). It happens frequently to people who are on their feet all day, such as mail carriers, or engaged in athletics, especially in running and jumping. The pain of plantar fasciitis is usually most significant in the morning, just after you get up from bed and begin to walk. Rest, orthotics, night splints, and antiinflammatory medications are employed in treatment. A Morton's neuroma is a painful lesion of the neural interconnection of the medial and lateral plantar nerves between the third and fourth toes. An eversion sprain of the ankle can break the medial malleolus or tear the deltoid ligament. An inversion sprain commonly injures the fibulocalcaneal ligament or anterior talofibular ligament.

A 34-year-old male distance runner visits the outpatient clinic with a complaint of pain he has suffered in his foot for the past week. The clinical examination indicates that the patient has an inflammation of the tough band of tissue stretching from the calcaneus to the ball of the foot. Which of the following conditions is most characteristic of these symptoms? A. Morton's neuroma B. Ankle eversion sprain C. Tarsal tunnel syndrome D. Plantar fasciitis E. Inversion sprain of the ankle

B. The deep fibular (peroneal) nerve supplies the dorsiflexors of the foot, including the extensor hallucis longus and extensor digitorum longus. It also supplies the tibialis anterior, an invertor of the foot. This nerve has sensory distribution only to the skin between the first two toes. The common fibular (peroneal) nerve supplies not only the preceding muscles but also the evertors of the foot and provides sensation for most of the dorsum of the foot. The sciatic nerve innervates the muscles of the posterior thigh and all muscles of the leg and foot, in addition to providing sensory supply in those areas. The superficial fibular (peroneal) nerve innervates the evertors of the foot and provides sensation for the dorsum of the foot. The tibial nerve is the nerve for muscles of the posterior compartment of the leg and also of the plantar region and supplies sensation over the medial aspect of the leg posteriorly and the plantar surface of the foot and toes

A 34-year-old male long-distance runner complained to the team physician of swelling and pain of his shin. Skin testing in a physical examination showed normal cutaneous sensation of the leg. Muscular strength tests showed marked weakness of dorsiflexion and impaired inversion of the foot. Which nerve serves the muscles involved in the painful swelling? A. Common fibular (peroneal) B. Deep fibular (peroneal) C. Sciatic D. Superficial fibular (peroneal) E. Tibial

C. When the popliteus contracts, it rotates the distal portion of the femur in a lateral direction. It also draws the lateral meniscus posteriorly, thereby protecting this cartilage as the distal femoral condyle glides and rolls backward, as the knee is flexed. This allows the knee to flex and therefore serves in unlocking the knee. The biceps femoris is a strong flexor of the leg and laterally rotates the knee when it is in a position of flexion. The gastrocnemius is a powerful plantar flexor of the foot. The semimembranosus, similar to the biceps femoris, is a component of the hamstring muscles and is involved in extending the thigh and flexing the leg at the knee joint. The rectus femoris is the strongest quadriceps muscle in extending the leg at the knee.

A 34-year-old male power lifter visits the outpatient clinic because he has difficulty walking. During physical examination it is observed that the patient has a problem unlocking the knee joint to permit flexion of the leg. Which of the following muscles is most likely damaged? A. Biceps femoris B. Gastrocnemius C. Popliteus D. Semimembranosus E. Rectus femoris

D. Plantar fasciitis is a common clinical condition that results from tearing or inflammation of the tough band of tissue stretching from the calcaneus to the ball of the foot (the plantar aponeurosis). It usually happens to people who are on their feet frequently or engaged in athletics, especially running and jumping. Plantar fasciitis is usually most painful in the morning, just after getting up from bed and beginning to walk. Rest, orthotics, night splints, and antiinflammatory medications are employed in treatment. A Pott's fracture is a bimalleolar fracture, specifically a fracture of the distal end of the fibula (lateral malleolus) and medial malleolus, with outward displacement of the foot. Dupuytren's fracture involves fracture of the distal fibula with dislocation of the foot. Each of these fractures occurs due to sudden and forceful eversion of the foot.

A 34-year-old male runner visits the outpatient clinic complaining of pain in his foot for the past week. Physical examination reveals inflammation of the tough band of tissue stretching from the calcaneus to the ball of the foot. Which of the following conditions is characteristic of these symptoms? A. Pott's fracture B. Dupuytren fracture C. Tarsal tunnel D. Plantar fasciitis E. Rupture of spring ligament

A. In a femoral hernia, abdominal contents are forced through the femoral ring, which is just lateral to the lacunar ligament (of Gimbernat) and just medial to the femoral vein. The femoral vein would be found immediately lateral to the femoral hernia. This is correct in most cases because in the majority of people, the femoral vein is found more medial to both the femoral artery and nerve in the femoral triangle. The adductor longus muscle as well as the pectineus muscle would be found deep and medial to the hernia.

A 34-year-old man is lifting heavy weights while doing squats. Unfortunately, while making a maximal effort, he drops the weight and immediately grabs at his upper thigh, writhing in pain. The man is admitted to the emergency department and during physical examination is diagnosed with a femoral hernia. What reference structure would be found immediately lateral to the herniated structures? A. Femoral vein B. Femoral artery C. Pectineus muscle D. Femoral nerve E. Adductor longus muscle

D. Bipartite patella is a normal variant of an unfused superolateral secondary ossification center, which can easily be mistaken for a fracture on a radiograph. The subcutaneous prepatellar bursa can become painfully enlarged with acute or chronic compression, as in crawling about on the knees. Osgood-Schlatter disease is painful involvement of the patellar ligament on the tibial tuberosity, commonly in children 10 to 14 years of age. The medial retinaculum is an expanded portion of the vastus medialis tendon toward the patella.

A 34-year-old man visits the outpatient clinic for an annual checkup. A radiographic examination of his knees is shown in Fig. 5-11. Physical examination reveals no pathology or pain to his knees. The patient has no past history of any knee problems. What is the most likely diagnosis? A. Enlarged prepatellar bursa B. Osgood-Schlatter disease C. Normal intercondylar eminence D. Bipartite patella E. Injury to lateral meniscus

B. The lower portion of the vastus medialis inserts upon the medial aspect of the patella and draws it medially, especially in the last quarter of extension—during which it is especially palpable in contraction. This lower portion of the muscle is referred to as the vastus medialis obliquus (VMO). Increasing the strength of this muscle can lessen the lateral dislocation of the patella. The rectus femoris arises from the anterior inferior iliac spine and lip of the acetabulum and draws the patella vertically upward, as does the vastus intermedius.

A 34-year-old woman has a direct blow to the patella by the dashboard of the vehicle during an automobile crash. The woman is admitted to the emergency department and radiologic examination reveals patellofemoral syndrome. This type of syndrome is characterized by lateral dislocation of the patella. Which of the following muscles requires strengthening by physical rehabilitation to prevent future dislocation of the patella? A. Vastus lateralis B. Vastus medialis C. Vastus intermedius D. Rectus femoris E. Patellar ligament

A. The talocrural (tibiotalar, ankle) joint is a hinge-type synovial joint between the tibia and talus. It permits dorsiflexion and plantar flexion, and fracture of this joint would affect these movements

A 37-year-old man is admitted to the hospital after an injury to his foot while playing flag football with friends on a Saturday morning. A series of radiographs demonstrates a fracture involving the talocrural (tibiotalar, ankle) joint. Which movements are the major ones to be affected by this injury? A. Plantar flexion and dorsiflexion B. Inversion and eversion C. Plantar flexion, dorsiflexion, inversion, and eversion D. Plantar flexion and inversion E. Dorsiflexion and eversion

B. The lateral femoral circumflex artery is a branch of the femoral artery close to the hip joint. It gives a branch that runs down the lateral aspect of the thigh and joins the genicular anastomosis via the superior lateral genicular artery. The medial circumflex femoral artery does not provide any branches that descend toward the knee. The anterior and posterior tibial arteries are the terminal branches of the popliteal artery and would not receive any blood if the popliteal is damaged. The fibular (peroneal) artery is a branch of the posterior tibial artery.

A 37-year-old unconscious man is rushed to the emergency department after being retrieved from a motor vehicle crash. On physical examination bruising and obvious deformity is seen over his left knee joint. Radiological studies showed a posteriorly dislocated supracondylar fracture with severe compression of the popliteal artery. Which of the following arteries would ensure adequate blood supply to the leg and foot in this patient? A. Medial femoral circumflex B. Lateral femoral circumflex C. Anterior tibial artery D. Posterior tibial artery E. Fibular (peroneal) artery

A. The tendon of the obturator internus leaves the pelvic cavity by passing through the lesser sciatic foramen, wrapping around the lesser sciatic notch, changing direction by about 90 degrees. It is joined there by the superior and inferior gemelli and inserts with them on the upper portion of the greater trochanter. The obturator externus arises on the external surface of the pubic bone and obturator membrane and inserts on the greater trochanter. The quadratus femoris arises from the ischial tuberosity and inserts on the intertrochanteric line of the femur. The gluteus medius and minimus insert together on the lateral aspect of the greater trochanter.

A 37-year-old woman had been suffering for months from piriformis entrapment syndrome, which was not relieved by physical therapy. Part of the sciatic nerve passed through the piriformis, and a decision was made for surgical resection of the muscle. When the area of entrapment was identified and cleared, a tendon could be seen emerging through the lesser sciatic foramen, at first hidden by two smaller muscles and several nerves and vessels destined for the region of the perineum. The tendons of which of the following muscles pass through this opening? A. Obturator internus B. Obturator externus C. Quadratus femoris D. Gluteus minimus E. Gluteus medius

A. Morton's neuroma most commonly involves compression (and possible enlargement) of an anastomosing branch that connects the medial and lateral plantar nerves between the third and fourth toes. The pain can be severe. The medial plantar nerve provides sensation for the medial three and a half toes; the lateral plantar nerve supplies the little toe and half of the fourth toe. The neural interconnection can be compressed between the transverse metatarsal ligament and the floor. Women are 10 times more likely than men to be afflicted with this problem, most likely due to wearing shoes that put excessive stress on the forefoot. In about 80% of cases the pain can be eased with different (less confining) shoes or cortisone injections.

A 39-year-old woman is admitted to the emergency department with a painful foot. Radiologic examination reveals a Morton's neuroma. What is the most typical location of this neuroma? A. Between the third and fourth metatarsophalangeal joints B. Between the second and third metatarsophalangeal joints C. Between the first and second metatarsophalangeal joints D. Between the fourth and fifth metatarsophalangeal joints E. In the region of the second, third, and fourth metatarsophalangeal joints

A. Any superficial inflammation in the gluteal region drains into the superficial horizontal group of inguinal nodes. The vertical group drains the lower limbs, whereas deep gluteal injuries drain into the superior and inferior gluteal nodes.

A 39-year-old woman who is a school teacher unwittingly sits on a thumbtack a student placed on her chair. Her left buttock becomes painful and inflamed. Which group of nodes will first receive lymph from the infected wound? A. Superficial horizontal group B. Superficial vertical group C. Superior and inferior gluteal nodes D. External iliac E. Deep inguinal

D. A positive valgus stress test indicates injury to the medial collateral ligament. Injuries to this ligament usually involve the anterior cruciate ligament. The femur is usually pushed posteriorly during stair climbing, an action that is opposed by a normal anterior cruciate ligament. Injury to the anterior cruciate ligament results in posterior displacement of the femur in relation to the tibia with difficulty climbing stairs. Extension of the knee is done mainly by the quadriceps femoris muscle. The posterior thigh muscles provide flexion of the knee. Gravity pushes the femur forward while walking down a flight, which is stabilized by the posterior cruciate ligament, which is not damaged in this case. The patient has no difficulty walking down the stairs; the posterior cruciate ligament stabilizes the knee during this action.

A 40-year-old man was brought to the emergency department after being struck by a car. He complained of pain to the left knee and leg and inability to bear weight on the affected limb. On examination, there was a joint effusion of the left knee, and tenderness over the medial and lateral side of the joint. A valgus stress test was positive while the varus stress test was negative. An MRI of the left knee showed complete disruption of multiple ligament support structures of the knee. What other symptoms are most likely possible in this patient? A. Inability to extend the knee B. Inability to flex the knee C. Instability of the knee when walking down a flight of stairs D. Instability of the knee when walking up a flight of stairs E. Excessive extension of the knee and difficulty walking down stairs

A. The plantar calcaneonavicular (spring) ligament supports the head of the talus and maintains the longitudinal arch of the foot. A fracture of the cuboid bone would not disrupt the longitudinal arch of the foot. Interruption of the plantar aponeurosis is not the best answer because this aponeurosis provides only passive support, unlike the spring ligament. A sprain of the anterior talofibular ligament would result from an inversion injury of the ankle and would not disrupt the longitudinal arch of the foot. A sprain of the deltoid ligament results from eversion of the ankle joint and would not disrupt the longitudinal arch of the foot.

A 41-year-old man is admitted to the emergency department with a swollen and painful foot. Radiologic examination reveals that the head of the talus has become displaced inferiorly, thereby causing the medial longitudinal arch of the foot to fall. What would be the most likely cause in this case? A. Tearing of the plantar calcaneonavicular (spring) ligament B. Fracture of the cuboid bone C. Interruption of the plantar aponeurosis D. Sprain of the anterior talofibular ligament E. Sprain of the deltoid ligament

49 D. The second perforating branch of the profunda femoris (deep femoral) artery commonly provides the nutrient artery to the femur, a vessel that passes through a rather large foramen to enter the proximal part of the shaft. The deep circumflex branch of the external iliac passes around the medial aspect of the iliac crest, also supplying the lower lateral part of the anterior abdominal wall. The acetabular branch of the obturator artery supplies tissues in the hip socket, usually including a branch to the ligament of the head of the femur. The lateral circumflex femoral branch of the deep femoral artery supplies the vastus lateralis muscle. The medial circumflex femoral branch of the deep femoral artery supplies proximal adductor musculature and the region of the hip joint, including the neck and head of the femur

A 42-year-old male sign painter is admitted to the emergency department after falling to the sidewalk from his ladder. Radiologic examination reveals a fracture of the proximal femur. Which of the following arteries supplies the proximal part of the femur? A. Deep circumflex iliac B. Acetabular branch of obturator C. Lateral circumflex femoral D. A branch of profunda femoris E. Medial circumflex femoral

A. The lumbosacral trunk consists of fibers from a portion of the ventral ramus of L4 and all of the ventral ramus of L5 and provides continuity between the lumbar and sacral plexuses. The deep fibular (peroneal) nerve receives supply from segments of L4, L5, and S1. It supplies the extensor hallucis longus and extensor digitorum longus, the main functions of which are extension of the toes and dorsiflexion of the ankle. L5 is responsible for cutaneous innervation of the dorsum of the foot. Injury to L4 would affect foot inversion by the tibialis anterior. Injury to L4 in the lumbosacral trunk would not affect the patellar tendon reflex, for these fibers are delivered by the femoral nerve. Therefore, an injury to the lumbosacral trunk would result in all of the patient's symptoms. Nerve root injury at L5 and S1 would result in loss of sensation of the plantar aspect of the foot and motor loss of plantar flexion, with weakness of hip extension and abduction. The fibularis (peroneus) longus and brevis are supplied by the superficial fibular (peroneal) nerve, which is composed of fibers from segments L5, S1, and S2; these are responsible for eversion of the foot (especially S1). Transection of the fibular (peroneal) division of the sciatic nerve would result in loss of function of all the muscles of the anterior and lateral compartments of the leg. Injury to the sciatic nerve will affect hamstring muscles and all of the muscles below the knee. Injury to the tibial nerve causes loss of plantar flexion and impaired inversion.

A 42-year-old man is admitted to the emergency department after his automobile hit a tree. He is treated for a pelvic fracture and several deep lacerations. Physi- cal examination reveals that dorsiflexion and inversion of the left foot and extension of the big toe are very weak. Sensation from the dorsum of the foot, skin of the sole, and the lateral aspect of the foot has been lost and the patellar reflex is normal. The foot is everted and plantar flexed. Which of the following structures is most likely injured? A. The lumbosacral trunk at the linea terminalis B. L5 and S1 spinal nerves torn at the intervertebral foramen C. Fibular (peroneal) division of the sciatic nerve at the neck of the fibula D. Sciatic nerve injury at the greater sciatic foramen ("doorway to the gluteal region") E. Tibial nerve in the popliteal fossa

B. The superficial inguinal nodes are located near the saphenofemoral junction and drain the superior thigh region. The vertical group receives lymph from the superficial thigh, and the horizontal group receives lymph from the gluteal regions and the anterolateral abdominal wall. The deep inguinal lie deep to the fascia lata and receive lymph from deep lymph vessels (popliteal nodes). The external and internal iliac nodes first receive lymph from pelvic and perineal structures.

A 42-year-old man is bitten on his posterior thigh by a dog. The superficial wound is sutured in the emergency department. Four days later the patient returns to the hospital with high fever and swollen lymph nodes. Which group of nodes first receives lymph from the infected wound? A. External iliac B. Vertical group of superficial inguinal C. Deep inguinal D. Horizontal group of superficial inguinal E. Internal iliac

D. The ligament of the head of the femur conveys a small blood vessel for supply of the head of the femur (primarily in childhood). The ligament is stretched during abduction and lateral rotation of the hip joint and has an important role in stabilizing an infant's hip joint before walking. It has the potential to increase stability of the joint in hip reconstruction in developmental hip dysplasia in the pediatric population. The strength of this ligament is comparable to the anterior cruciate ligament of the knee. The iliofemoral ligament (the inverted "Y-shaped ligament of Bigelow") on the anterior aspect of the hip bone resists hyperextension of the hip joint. The pubofemoral ligament arises from the pubic bone and is located on the inferior side of the hip joint; it resists abduction of the joint. The ischiofemoral ligament is a triangular band of strong fibers that arises from the ischium and winds upward and laterally over the femoral neck, strengthening the capsule posteriorly. The transverse acetabular ligament attaches to the margins of the acetabular notch and provides origin for the ligament of the head of the femur. The transverse acetabular ligament is fibrous, not cartilaginous, but is regarded as part of the acetabular labrum.

A 42-year-old mother of three children visits the outpatient clinic complaining that her youngest son cannot walk yet even though he is 4 years old. Radiologic and physical examinations reveal an unstable hip joint. Which of the following ligaments is responsible for stabilization of the hip joint in childhood? A. Iliofemoral B. Pubofemoral C. Ischiofemoral D. Ligament of the head of the femur E. Transverse acetabular ligament

E. The vastus lateralis muscle is located on the lateral aspect of the thigh. The distal portion of this muscle lies superficial to the proximal part of the lateral aspect of the joint capsule of the knee. When a needle is inserted superiorly and laterally to the patella, it penetrates the vastus lateralis muscle on its course to the internal capsule. The short head of biceps femoris has its origin on the posterior aspect of the femur, merges with the long head of the biceps femoris, and inserts on the head of the fibula. The rectus femoris passes longitudinally on the medial aspect of the femur and inserts on the tibial tuberosity, via the patellar tendon, or quadriceps tendon. A needle inserted laterally to the patella would not penetrate this muscle. The sartorius originates on the anterior superior iliac spine and forms part of the pes anserinus, which inserts on the medial aspect of the proximal part of the tibia. A needle inserted laterally to the patella would not penetrate this muscle.

A 43-year-old man visits the outpatient clinic with a painful, swollen knee joint. The patient's history reveals chronic gonococcal arthritis. A knee aspiration is ordered for bacterial culture of the synovial fluid. A standard suprapatellar approach is used, and the needle passes from the lateral aspect of the thigh into the region immediately proximal to and deep to the patella. Through which of the following muscles would the needle pass? A. Adductor magnus B. Short head of biceps femoris C. Rectus femoris D. Sartorius E. Vastus lateralis

A. Gracilis due to its shape, size, and more importantly the nature of neurovascular supply is used very commonly in reconstructive surgery as a free functioning autograft. Also, the other adductors of the thigh compensate for the absence of the gracilis. For similar reasons the remaining muscles are not good candidates during reconstructive surgery of the upper limb.

A 43-year-old victim of a drunk driving car crash is undergoing reconstructive arm surgery. The surgeon performs an autograft using a weak adductor of the leg located superficially on the medial side of the thigh. Which muscle is most likely being harvested to perform this reconstruction? A. Gracilis B. Sartorius C. Rectus femoris D. Vastus lateralis E. Vastus medialis

C. Entrapment compression of all or part of the sciatic nerve by the piriformis can mimic disc herniation, most commonly resembling compression of spinal nerve S1. This results in pain down the posterior aspect of the thigh and leg and the lateral side of the foot. In this case, loss of sensation over the dorsum of the foot and weakness of foot extension, in addition to eversion, indicate that more than S1 is involved. Foot drop would be anticipated with fibular (peroneal) nerve involvement. As noted also in a previous question, compression of the common fibular (peroneal) division of the sciatic nerve by the piriformis gives rise to the clinical condition known as piriformis entrapment. This condition is associated with point pain in the gluteal area, pain in the posterior part of the limb, and possible weakness of muscles in the lateral and anterior compartments of the leg. It can be confused with herniated disc (L5) compression of S1 and sciatica. Paralysis of plantar flexion occurs with a lesion of the tibial division of the sciatic nerve or the tibial nerve. Paralysis of the quadriceps is associated with pathology of the femoral nerve. Clonic contraction of the adductors could result from obturator nerve problems

A 43-year-old woman is examined by a neurologist, to whom she complains of pain in her lower limb of 6 months' duration. She has pain in the gluteal area, thigh, and leg. The neurologist observes reduced sensation over the dorsum and lateral side of the involved foot and some weakness in foot dorsiflexion and eversion. A diagnosis of a piriformis entrapment syndrome is made, with compression of the fibular (peroneal) division of the sciatic nerve. Which of the following conditions did the neurologist also most likely find during her physical examination of the patient? A. Paralysis of plantar flexion B. Instability of the knee, due to paralysis of the quadriceps femoris C. Foot drop D. Spasm or clonic contractures of the adductor musculature of the thigh E. Loss of sensation in the gluteal area, by paralysis of anterior cluneal nerves

A. The gluteal region (buttocks) is a common site for intramuscular injection of drugs, particularly if the volume of the injection is large. To avoid injury to the underlying sciatic nerve, the injection should be given well forward on the upper outer quadrant of the buttock (superolateral quadrant). The patient is showing the Trendelenburg gait pattern (or gluteus medius lurch), which is caused by weakness of the gluteus medius and minimus muscles. These muscles are supplied by the superior gluteal nerve (L4, L5, S1), which emerges from the greater sciatic notch above the upper border of the piriformis and immediately disappears beneath the posterior border of the gluteus medius and runs forward between the gluteus medius and minimus. Intramuscular injection in the upper inner quadrant (superomedially) is most likely to damage this nerve. The sciatic nerve is most likely damaged in the inferomedial quadrant of the buttock.

A 43-year-old woman receives deep intramuscular injections for the past week for treatment of a sexually transmitted disease. She complains to her doctor that she has difficulty walking. During physical examination her right hip drops every time she raises her right foot. Which of the following injection locations will most likely correspond with the clinical presentation of this patient? A. Superomedial quadrant of the buttock B. Superolateral quadrant of the buttock C. Inferomedial quadrant of the buttock D. Inferolateral quadrant of the buttock E. Posterior thigh

B. The obturator nerve innervates the adductor muscles, including the gracilis, pectineus, and obturator externus. The tibial nerve supplies the calf muscles and intrinsic muscles in the plantar portion of the foot. The inferior gluteal nerve innervates the gluteus maximus; the superior gluteal nerve supplies the gluteus medius and minimus and tensor fasciae latae. The femoral nerve provides motor supply to the quadriceps femoris, sartorius, and, in some cases, the pectineus. This gait pattern is characteristic of hypertonia in the lower limb. As a result these areas become flexed to various degrees, giving the appearance of crouching, while tight adductors produce extreme adduction

A 45-year-old intoxicated man was struck by a tour bus while walking in the middle of the street. The man was admitted to the emergency department and during physical examination was diagnosed with "adductor gait," in which an individual crosses one limb in front of the other, due to powerful hip adduction. Which of the following nerves was most likely involved in this condition? A. Tibial B. Obturator C. Inferior gluteal D. Superior gluteal E. Femoral

B. The posterior cruciate ligament tightens in flexion of the knee. It can be damaged by posterior displacement of the tibia upon the femur. With the patient seated, a rupture of the ligament can be demonstrated by the ability to push the tibia posteriorly under the femur. This is called the posterior drawer sign because it's similar to pushing in a desk drawer. The anterior cruciate ligament resists knee hyperextension. The lateral collateral ligament is a thick, cordlike band that passes from the lateral femoral condyle to the head of the fibula. It is located external to the capsule of the knee joint. The lateral meniscus is a nearly circular band of fibrocartilage that is located laterally within the knee joint. It is less frequently injured than the medial meniscus because it is not attached to the joint capsule or other ligaments. The patellar ligament is the heavy, ligamentous band of insertion of the quadriceps muscle to the tibial tuberosity. GAS 584-5

A 45-year-old is admitted to the hospital after his left leg impacted a fence post when he was thrown from a powerful four-wheel all-terrain vehicle. Radiologic examination reveals posterior displacement of the tibia upon the femur. Which of the following structures was most likely injured? A. Anterior cruciate ligament B. Posterior cruciate ligament C. Lateral collateral ligament D. Lateral meniscus ligament E. Patellar ligament

A. All the branches of the posterior femoral cutaneous nerve are cutaneous. It arises from the dorsal divisions of the first and second and the ventral divisions of the second and third sacral nerves and travels through the greater sciatic foramen beneath the piriformis muscle to innervate the shin over the lower parts of the gluteus maximus muscle through the inferior clunial nerves and the posterior surface of the thigh and leg and perineum via its perineal branches. The lateral femoral cutaneous nerve, which innervates the lateral surface of the thigh, is an incorrect choice; the obturator, sciatic, and femoral nerves all have both motor and sensory branches and can be eliminated, since no motor deficits were described.

A 45-year-old man after being diagnosed with a posterior acetabular fracture is taken to the operating room to repair the fracture. During the neurological examination the physician notices loss of sensation to the skin of the inferior half of the buttocks, posterior, and upper medial thigh. The patient had a normal neurovascular examination preoperatively. Which of the following nerves was mostly likely damaged during the operation? A. Posterior femoral cutaneous B. Obturator nerve C. Sciatic D. Femoral E. Lateral femoral cutaneous

E. The deep fibular (peroneal) nerve along with the superficial fibular (peroneal) nerve are branches of the common fibular nerve. The deep fibular (peroneal) nerve innervates muscles of the anterior compartment of the leg dorsiflexors of the foot and the skin between the great toe and second toes, while the superficial fibular nerve innervates the lateral compartment muscles of the leg, which are evertors of the foot and the skin on most of the dorsum of the foot. If the common fibular nerve were damaged all the structures that receive innervation via this nerve will be compromising dorsiflexion and eversion. Damage to the superficial fibular nerve affects the ability to evert the foot but does not result in foot drop, making the deep fibular nerve, which innervates the dorsiflexors of the foot, the best choice as the injury describes foot drop with the ability to evert the foot conserved. The saphenous nerve is a cutaneous nerve, while the tibial nerve innervates posterior compartment muscles.

A 45-year-old man is admitted to the emergency department after a fall and subsequent leg injury. On physical examination the patient has a foot drop but eversion is unaffected. Which nerve is most likely injured? A. Tibial B. Common fibular (peroneal) C. Superficial fibular (peroneal) D. Saphenous E. Deep fibular (peroneal)

C. The Achilles tendon reflex is a function of the triceps surae muscle, composed of the gastrocnemius and soleus muscles that insert on the calcaneus. The innervation is provided primarily by spinal nerve S1. The S1 root leaves the vertebral column at the S1 foramen of the sacrum, but a herniated disc at the L5-S1 intervertebral space puts the S1 root under tension, resulting in pain and possible weakness or paralysis of S1 supplied muscles, especially the plantar flexors. A disc lesion at L3-4 would affect the L4 spinal nerve (affecting foot inversion and extension); a lesion at L4-5 would cause problems with L5 (hip abduction and knee flexion). A disc lesion at S1-2 in the sacrum is improbable, unless there was lumbarization of the S1 vertebra. The gluteal crush syndrome usually occurs when a patient has been lying unconscious and unmoving on a hard surface for an extended period of time.

A 45-year-old man is admitted to the emergency department after experiencing a sharp pain while lifting a box of books. He told the physician that he "felt the pain in my backside, the back of my thigh, my leg, and the side of my foot." During physical examination it is observed that his Achilles tendon jerk is weakened on the affected side. Which is the most likely cause of injury? A. Disc lesion at L3-4 B. Disc lesion at L4-5 C. Disc lesion at L5-S1 D. Disc lesion at S1-2 E. Gluteal crush syndrome of the sciatic nerve or piriformis syndrome

E. The deep fibular (peroneal) nerve is responsible for innervating the muscles of the anterior compartment of the leg, which are responsible for toe extension, foot dorsiflexion, and inversion. Injury to this nerve will result in foot drop and also loss of sensation between the first and second toes. Injury to the tibial nerve affects the posterior compartment muscles of the leg, which are responsible for plantar flexion and toe flexion, as well as the intrinsic muscles of the sole of the foot. The common fibular (peroneal) nerve splits into the superficial and deep fibular (peroneal) nerves, and these supply both the lateral and anterior compartments. The superficial fibular (peroneal) nerve innervates the fibularis (peroneus) longus and brevis muscles, which provide eversion of the foot. If the common fibular (peroneal) nerve were injured, eversion of the foot and plantar flexion would be lost in addition to dorsiflexion and inversion. The saphenous nerve, a continuation of the femoral nerve, is a cutaneous nerve that supplies the medial side of the leg and foot and provides no motor innervation.

A 45-year-old man is treated at the hospital after he fell from his bicycle. Radiologic examination reveals fractures both of the tibia and the fibula. On physical examination the patient has a foot drop, but normal eversion (Fig. 5-1). Which of the following nerves is most likely injured? A. Tibial B. Common fibular (peroneal) C. Superficial fibular (peroneal) D. Saphenous E. Deep fibular (peroneal)

E. The tibial nerve is responsible for innervating the posterior compartment of the leg. These muscles are responsible for knee flexion, plantar flexion, and intrinsic muscle functions of the foot. Compression of this nerve can affect plantar flexion of the foot. Dorsiflexion of the foot would be compromised if the deep fibular (peroneal) nerve were compressed by this Baker's cyst. Flexion of the thigh is a function of muscles supplied by lumbar nerves and the femoral nerve. The deep fibular (peroneal) nerve is also responsible for extension of the digits, whereas the femoral nerve is responsible for extension of the leg.

A 45-year-old man presents at the local emergency clinic with the complaint of a painful knee and difficulty in walking. A computed tomography (CT) scan examination reveals a very large cyst in the popliteal fossa compressing the tibial nerve. Which movement will most likely be affected? A. Dorsiflexion of the foot B. Flexion of the thigh C. Extension of the digits D. Extension of the leg E. Plantar flexion of the foot

C. The lateral plantar nerve innervates the interossei and adductor hallucis. These losses would be obvious when the patient attempts to abduct and adduct the toes. Sensation would be absent over the lateral side of the sole, the fifth and fourth toes, and half of the third toe. The medial plantar nerve provides sensation over the plantar surface of the first and second toes and half of the third toe as well as function of the so-called LAFF muscles: first lumbrical, abductor hallucis, flexor hallucis brevis, and flexor digitorum brevis

A 46-year-old woman stepped on a broken wine bottle on the sidewalk and the sharp glass entered the posterior part of her foot. The patient was admitted to the hospital, and a physical examination concluded that her lateral plantar nerve had been transected (cut through). Which of the following conditions will most likely be confirmed by further physical examination? A. Loss of sensation over the plantar surface of the third toe B. Paralysis of the abductor hallucis C. Paralysis of the interossei and adductor hallucis D. Flexor hallucis brevis paralysis E. Flexor digitorum brevis paralysis

D. The S1 nerve root provides cutaneous innervation to the lateral aspect of the ankle, the lateral sides of the dorsum and sole of foot, and motor innervation to the gastrocnemius muscle, which plantar flexes the foot and contracts during the ankle jerk reflex. It receives its innervation from the S1, S2 nerve roots via the tibial nerve, making D the correct choice. T12 roots do not reach the foot; L2 roots will reach the hip region and thigh; L4 innervates the invertors of the foot and skin over medial leg, ankle, and side of foot; and S3 innervates the sitting area of the buttocks, posterior scrotum or labia, and the small muscles of the foot.

A 48-year-old man comes to the physician because of severe back pain for 2 days. The pain radiates down to the buttock, posterior thigh, and posterolateral leg. He also has numbness on the lateral side of his left foot. On physical examination sensation to pain is decreased over the lateral side of the left foot. Deep tendon reflexes are absent at the left ankle, and there is a weakness of dorsiflexion of the left foot. Compression of which of the following nerve roots is the most likely cause of these findings? A. T12 B. L2 C. L4 D. S1 E. S3

D. Femoral artery puncture is one of the most common vascular procedures. The femoral artery can be localized often by simply feeling for the strongest point of the femoral pulse just inferior to the inguinal ligament. The femoral artery can be accessed with fluoroscopic assistance at the medial edge of the upper portion of the head of the femur. It is easily localized by Doppler ultrasound if the pulse is difficult to detect, such as in an obese patient. It is here that catheters are passed into the femoral artery for catheterization of abdominopelvic and thoracic structures and for antegrade angiography. It is also a site where arterial blood can be obtained for gas analysis. The mid inguinal point, halfway between the anterior superior iliac spine and the pubic symphysis, can be either medial or lateral to the femoral artery and is not a dependable landmark. A needle inserted at the level of the inguinal crease, or inferior to the femoral head, can enter the femoral artery distal to the origin of the deep femoral artery, presenting more risk for accidental vascular injury. Four centimeters lateral to the pubic tubercle overlies the deep inguinal ring, with potential entry to spermatic cord, femoral vein, or artery. The fossa ovalis is the opening in the deep fascia of the thigh for the termination of the great saphenous vein in the femoral vein.

A 48-year-old woman is admitted to the hospital with severe abdominal pain. Several imaging methods reveal that the patient suffers from intestinal ischemia. An abdominopelvic catheterization is ordered for antegrade angiography. A femoral puncture is performed. What is the landmark for femoral artery puncture? A. Halfway between anterior superior iliac spine and pubic symphysis B. 4.5 cm lateral to the pubic tubercle C. Midpoint of the inguinal skin crease D. Medial aspect of femoral head E. Lateral to the fossa ovalis

A. The femoral ring is the abdominal opening of the femoral canal. A femoral hernia passes through the femoral ring into the femoral canal deep and inferior to the inguinal ligament. It can appear as a bulging at the saphenous hiatus (fossa ovalis) of the deep fascia of the thigh, the hiatus through which the saphenous vein passes to the femoral vein. The superficial inguinal ring is the triangular opening in the aponeurosis of the external abdominal oblique and lies lateral to the pubic tubercle. The deep inguinal ring lies in the transversalis fascia lateral to the inferior epigastric vessels. Herniation into either of these two openings is associated with an inguinal hernia. The obturator canal, a bony opening between the superior and inferior ramus of the pubic bone, is the site of an obturator hernia.

A 49-year-old male construction worker is admitted to the emergency department with a painful lump on the proximal medial aspect of his thigh. Radiologic and physical examinations reveal that the patient has a herniation of abdominal viscera beneath the inguinal ligament into the thigh. Through which of the following openings will a hernia of this type initially pass to extend from the abdomen into the thigh? A. Femoral ring B. Superficial inguinal ring C. Deep inguinal ring D. Fossa ovalis E. Obturator canal

B. Contraction of the gastrocnemius on the fractured calcaneus would increase the pain because the gastrocnemius inserts with the soleus upon that bone, via the calcaneal tendon, or tendo Achilles. The flexor digitorum profundus passes the ankle medially to enter the sole of the foot, where it inserts upon the distal phalanges. The tibialis posterior, likewise, passes under the medial malleolus, with complex insertions upon the navicular bone, cuneiform bones, metatarsal bones, and the cuboid bone. The tibialis anterior, a muscle of the anterior leg compartment, inserts upon the navicular bone and, with the tibialis posterior, is a strong invertor of the foot. The fibularis (peroneus) longus is a muscle of the lateral compartment of the leg. It passes under the lateral malleolus, entering the sole of the foot by crossing the lateral surface of the calcaneus, and inserts primarily into the medial cuneiform and base of the first metatarsal bone.

A 49-year-old male worker fell from a ladder, with his weight impacting on the heels of his feet. Radiologic examination reveals comminuted calcaneal fractures. After the injury the contraction of which one of the following muscles would most likely increase the pain in the injured foot? A. Flexor digitorum profundus B. Gastrocnemius C. Tibialis posterior D. Tibialis anterior E. Fibularis (peroneus) longus

A. If the femoral artery is occluded, the descending branch of the lateral circumflex femoral will provide collateral circulation to the thigh. The descending genicular artery is a branch of the femoral and therefore would also be occluded. The medial circumflex femoral artery is a proximal branch of the deep femoral artery and supplies part of the head of the femur. The first perforating branch of the deep femoral artery supplies a small portion of the muscles of the posterior thigh. Finally, the obturator artery supplies a very small artery and vascularizes only the most proximal part of the head of the femur and usually only during the early years of life.

A 49-year-old man is admitted to the emergency department complaining that he has difficulties walking. Physical examination reveals that the patient suffers from peripheral vascular disease. An ultrasound examination reveals an occlusion of his femoral artery at the proximal portion of the adductor canal. Which of the following arteries will most likely provide collateral circulation to the thigh? A. Descending branch of the lateral circumflex femoral B. Descending genicular C. Medial circumflex femoral D. First perforating branch of deep femoral E. Obturator artery

D. The posterior tibial artery passes under the medial malleolus, about halfway between that bony landmark and the heel, or the calcaneus. The medial edge of the plantar aponeurosis can be palpated just medial to the muscular belly of the abductor hallucis. The sural nerve and the short (lesser) saphenous vein pass around the lateral side of the foot, about halfway between the lateral malleolus and the calcaneus. The sartorius passes behind the medial femoral condyle to insert on the proximal, medial aspect of the tibia via the pes anserinus; usually no pulse can be felt clearly there. The popliteal artery passes between the two heads of the gastrocnemius, where the arterial pulse may be felt very deeply, medial to the midline.

A 49-year-old man is admitted to the emergency department with a cold and pale foot. Physical examination reveals that the patient suffers from peripheral vascular disease; duplex ultrasound studies indicate possible occlusion of his popliteal artery, and the pulse of the posterior tibial artery is absent. What is the most common location for palpation of the pulse of the posterior tibial artery? A. Lateral to the muscular belly of the abductor hallucis B. Posteroinferior to the medial femoral condyle C. Groove midway between the lateral malleolus and the calcaneus D. Groove midway between the medial malleolus and the calcaneus E. Medially, between the two heads of the gastrocnemius

B. The femoral triangle is the best place to palpate the femoral pulse. It is bounded by the sartorius muscle laterally, adductor longus medially, and the inguinal ligament superiorly. It contains the femoral vein, artery, and nerve (from medial to lateral, respectively). The adductor canal lies deep between the anterior and medial compartments of the thigh and therefore cannot be palpated. The popliteal fossa is the fossa at the back of the knee and contains the popliteal artery and vein, tibial nerve, and common fibular (peroneal) nerve. The femoral pulse cannot be palpated here. The inguinal canal is in the pelvis and is in communication with the anterior abdominal wall. It contains the spermatic cord in males and round ligament of the uterus in females.

A 49-year-old man is admitted to the emergency department with a cold and pale foot. Physical examination reveals that the patient suffers from peripheral vascular disease; his popliteal artery is occluded and no pulse is felt upon palpation. What is the landmark to feel the pulse of the femoral artery? A. Adductor canal B. Femoral triangle C. Popliteal fossa D. Inguinal canal E. Pubic symphysis

D. The great saphenous vein is commonly used in coronary artery bypass grafts. Because branches of the saphenous nerve cross the vein in the distal part of the leg, the nerve can be damaged if the vein is stripped from the ankle to the knee. Stripping the vein in the opposite direction can protect the nerve and lessen the postoperative discomfort of patients. The saphenous nerve is responsible for cutaneous innervations on the medial surface of the leg and the medial side of the foot. Injury to this nerve will result in a loss of sensation and also can create chronic dysesthesias in the area. The common fibular (peroneal) nerve bifurcates at the neck of the fibula into the superficial and deep fibular (peroneal) nerves, which continue on to innervate the lateral and anterior compartments of the leg, respectively. These nerves are lateral and therefore not associated with the great saphenous vein. The lateral sural nerve is a cutaneous nerve that arises from the junction of branches from the common fibular (peroneal) nerve and tibial nerve and innervates the skin on the posterior aspect of the leg and lateral side of the foot. This nerve is often harvested for nerve grafts elsewhere in the body. The tibial nerve is a terminal branch of the sciatic nerve that continues deep in the posterior compartment of the leg.

A 49-year-old man underwent a coronary bypass graft procedure using the great saphenous vein. Postoperatively, the patient complains of pain and general lack of normal sensation on the medial surface of the leg and foot on the limb from which the graft was harvested. Which nerve was most likely injured during surgery? A. Common fibular (peroneal) B. Superficial fibular (peroneal) C. Lateral sural D. Saphenous E. Tibial

B. The child has the problem of talipes equinovarus, or clubfoot. Clubfoot is a congenital malformation observed in about 1 in 1000 pediatric patients and first appears in the first trimester of pregnancy. This syndrome combines plantar flexion, inversion, and adduction of the foot. The heel is drawn upward by the tendo calcaneus and turned inward; the forefoot is also adducted, or turned inward. The foot usually is smaller than normal. In coxa vara, the angle between the femoral shaft and neck is reduced to less than 120 degrees, often due to excessive activity of the adductor musculature. Hallux valgus is also known as bunion, in which the big toe points laterally. Hallux varus involves a medial deviation of the first metatarsal or big toe, sometimes the result of attempted correction of bunions. It can also result from arthritis or muscular problems.

A 5-month-old baby boy is admitted to the pediatric orthopedic clinic. During physical examination it is noted that the baby has inversion and adduction of the forefoot relative to the hindfoot and plantar flexion. Which of the following terms is diagnostic for the signs observed on physical examination? A. Coxa vara B. Talipes equinovarus C. Hallux valgus D. Hallux varus E. Plantar fasciitis

B. In infants and children up to about 8 years of age, the head of the femur gets its arterial supply by a direct branch of the obturator artery (variably, the medial circumflex femoral). The arterial supply reaches the head of the femur at the fovea capitis by traveling along the ligament of the head of the femur. Probably due to repeated torsion on the ligament, and therefore on the artery, this artery occludes early in life. In turn, this source of supply is replaced by branches of the gluteal and femoral circumflex vessels.

A 5-year-old boy is admitted to the emergency department after a car collision. Radiologic examination reveals a fracture of the head of the femur. An MRI examination reveals a large hematoma. Which of the following arteries is most likely injured? A. Deep circumflex iliac B. Acetabular branch of obturator C. Descending branch of lateral circumflex femoral D. Medial circumflex femoral E. Radicular branches of circumflex artery

A. Dorsalis pedis pulse is palpated at the prominent arch of the top of the foot between the first and second metatarsal bones between the tendon of the extensor hallucis longus and extensor digitorum longus for the second toe.

A 50-year-old diabetic man presents for a routine wellness checkup. During physical examination it is noted that he has paraesthesia in a classic glove and stocking distribution. The physician decides on a complete peripheral vascular system examination, which includes palpating the pulse of the dorsalis pedis. Where can the dorsalis pedis pulse be palpated? A. Between the tendons of extensor hallucis and extensor digitorum longus on the dorsum of the foot B. Superior to flexor hallucis longus just distal to the tarsal tunnel C. Inferolateral to the pubic symphysis and medial to the deep dorsal vein of the penis D. 2 cm anterior to the medial malleolus E. 2 cm posterior to the medial malleolus

A. When the anterior cruciate ligament is torn, the tibia can be slightly displaced anteriorly from the area of the knee joint by pulling firmly with both hands upon the leg, with the patient in a seated position. This is a positive anterior drawer sign.

A 50-year-old man is admitted to the emergency department after a car crash. An MRI examination reveals an injured anterior cruciate ligament. Physical examination reveals a positive drawer sign. Which of the following signs is expected to be present during physical examination? A. The tibia can be slightly displaced anteriorly B. The tibia can be slightly displaced posteriorly C. The fibula can be slightly displaced posteriorly D. The fibula can be slightly displaced anteriorly E. The tibia and fibula can be slightly displaced anteriorly

C. The lymphatic drainage of the leg is such that superficial lymphatics on the anterolateral side of the foot and leg and all the deep lymphatics in the foot and leg first drain into the popliteal nodes and then to the deep inguinal nodes. This patient has an infected anterolateral mid leg injury, which will first drain into the popliteal nodes. The vertical group of superficial inguinal nodes receives superficial lymphatics from the medial side of the foot, leg, and all the superficial lymph from the thigh. The horizontal group of superficial lymphatics receives lymph from the anterior abdominal wall below the umbilicus, perineum (except the glans penis in men and clitoris in women), and lower third of the anal canal. The popliteal nodes eventually drain to the deep inguinal nodes, but are usually not palpable. The iliac nodes are deep structures and are not be palpable during physical examination.

A 50-year-old woman is admitted to the emergency department complaining of painful swelling to the left leg, fever, and malaise for 2 days. The patient has a history of type 2 diabetes mellitus, and she was bitten on the left leg by an insect a week before presentation. She scratched the pruritic area and applied alcohol to the site when the swelling increased; a purulent fluid began to drain from it 2 days later. During physical examination, the patient was febrile (40° C) with 5 × 5 cm tender, fluctuant swelling over the anterolateral aspect of the middle third of the left leg, which drained copious amounts of purulent fluid. Which of the following findings is most likely to be also present during physical examination of this patient? A. Tender vertical group of superficial inguinal lymph nodes B. Enlarged horizontal group of superficial inguinal lymph nodes C. Enlarged group of deep inguinal lymph nodes D. Enlarged popliteal lymph nodes E. Enlarged iliac nodes

B. The psoas muscle arises from the base of the transverse processes, the sides of the vertebral bodies, and the intervertebral discs, from the twelfth thoracic to the fifth lumbar vertebrae and inserted into the lesser trochanter of the femur. The sheath of the psoas retains the pus of a psoas abscess, and spinal tuberculosis may present as a cold abscess in the groin (in the vicinity of the lesser trochanter). The psoas is enclosed in a fibrous sheath that is derived from the lumbar fascia. The sheath is not part of the lumbar fascia, but the lateral edge blends with the anterior layer of that fascia

A 51-year-old immigrant with tuberculosis is found to have large flocculent masses over the lateral lumbar spine. There is a similar mass located in the ipsilateral groin. Physical examination reveals increased tenderness just medial to the ipsilateral anterior superior iliac spine on palpation. This pattern of involvement most likely suggests an abscess tracking along which of the following muscles? A. Piriformis B. Psoas major C. Adductor longus D. Gluteus maximus E. Obturator internus

E. The talus can be rotated externally when the ankle sustains a trimalleolar fracture, also called a Henderson fracture. The fracture may be caused by eversion and posterior displacement of the talus. This injury involves the fracture of the distal fibula (lateral malleolus); the medial malleolus of the tibia; and the posterior portion, or lip, of the tibial plafond (the distal articular portion of the tibia, sometimes referred to as the posterior malleolus). The posterior part of the plafond is not truly a malleolus but acts this way in this type of twisting fracture of the ankle. The talus can be forced from its normal position in this fracture, adding to the instability of the ankle. The other bones listed are relatively far from the site of the fractures

A 52-year-old woman is admitted to the emergency department after severely injuring her right lower limb when she fell from a trampoline. Radiologic examination reveals a trimalleolar fracture of the ankle involving the lateral malleolus, medial malleolus, and the posterior process of the tibia. Which of the following bones will also most likely be affected? A. Navicular B. Calcaneus C. Cuneiform D. Cuboid E. Talus

B. The right superior gluteal nerve is the correct choice. When a person stands on one leg or walks, the gluteus medius, gluteus minimus, and tensor muscles of the fascia latae act in synergy to stabilize the hip joint by abducting the hip (pelvic tilt). These muscles receive their innervation from the superior gluteal nerve. The abductors of the hip, as they contract to maintain the stability of the hip joint, draw the pelvis forcefully toward the weigh-bearing leg, causing the opposite side of the pelvis to tilt in that same direction. The right superior gluteal nerve innervates its ipsilateral medius, minimus, and tensor muscles of the fascia latae. Loss of these muscles results in a positive Trendelenburg sign with the pelvis dropping on the left side.

A 53-year-old woman reports difficulty in walking. Physical examination showed a positive Trendelenburg sign when she is asked to stand on her right leg. Which nerve has been compromised to produce the positive sign? A. Sciatic B. Right superior gluteal C. Left inferior gluteal D. Left superior gluteal E. Right inferior gluteal

A. The "unhappy triad" (of O'Donoghue) is composed of the medial collateral ligament, medial meniscus, and anterior cruciate ligament. Sudden, forceful thrusts against the lateral side of the knee put tension on the medial collateral ligament, which can then rupture. The medial meniscus is attached to the medial collateral ligament so that it then tears. The anterior cruciate ligament resists hyperextension of the knee; thus, it is the third structure that ruptures in the "unhappy triad" of the knee.

A 55-year-old cowboy is admitted to the emergency department after he was knocked from his feet by a young longhorn steer. MRI examination reveals a large hematoma in the knee joint. Physical examination reveals that the patient suffers from the "unhappy triad" (of O'Donoghue). Which of the following structures are involved in such an injury? A. Medial collateral ligament, medial meniscus, and anterior cruciate ligament B. Lateral collateral ligament, lateral meniscus, and posterior cruciate ligament C. Medial collateral ligament, lateral meniscus, and anterior cruciate ligament D. Lateral collateral ligament, medial meniscus, and anterior cruciate ligament E. Medial collateral ligament, medial meniscus, and posterior cruciate ligament

D. The dorsalis pedis is the continuation of the anterior tibial artery into the foot, as it passes the distal end of the tibia and the ankle joint. The pulse of the dorsalis pedis can be felt between the tendon of the extensor hallucis longus and the tendon of the extensor digitorum longus to the second toe. A strong pulse is a positive indicator of circulation through the limb. The fibular (peroneal) artery is a branch of the posterior tibial artery and passes in the calf between the flexor hallucis longus and tibialis posterior, making it difficult to palpate. The deep plantar artery, the extension of the first dorsal interosseous or lateral plantar arteries, passes deep to the aponeurotic tissues and central muscles of the foot, making palpation unlikely. The dorsal metatarsal branches of the dorsalis pedis pass under cover of the extensor digitorum longus and brevis tendons. Palpable pulses of the first or other dorsal metatarsal arteries can therefore be difficult to detect

A 55-year-old man is admitted to the hospital for an iliofemoral bypass. The operation is performed successfully and the blood flow between the iliac and femoral arteries is restored. During rehabilitation which of the following arteries should be palpated to monitor good circulation of the lower limb? A. Anterior tibial B. Deep fibular (peroneal) C. Deep plantar D. Dorsalis pedis E. Dorsal metatarsal

A. Flat foot (pes planus) is due to flattening of the medial longitudinal arch. Often congenital, it may be associated with minor structural anomalies of the tarsal bones. This condition can be seen in wet footprints in which the medial surface of the sole (normally raised in an arch) is visible. Treatment may include intensive foot exercises or arch supports worn in the shoes. Occasionally, surgery is needed in the form of arthrodesis (fusion of the tarsal bones). Pes cavus is a deformity of the foot characterized by a very high medial arch and hyperextension of the toes. The long plantar ligament is a passive ligament of the longitudinal arch. The long plantar ligament connects the calcaneus and cuboid bones. It can be involved with the plantar aponeurosis in plantar fasciitis. The long plantar ligament converts the cuboid groove into a canal for the tendon of the fibularis (peroneus) longus. The deltoid ligament is a very strong ligament that interconnects the tibia with the navicular, calcaneus, and talus bones. The medial malleolus will usually fracture before this ligament will tear. The plantar calcaneonavicular, or spring, ligament is a key element in the medial longitudinal arch; it supports the head of the talus bone and thereby is subject to vertical forces exerted through the lower limbs. In the present case, the bilateral pes planus appears to be the res

A 55-year-old man visits the outpatient clinic complaining that he cannot walk more than 5 minutes without feeling severe pain in his feet. An image of the feet of this patient is shown in Fig. 5-10. What is the most common cause of this condition? A. Collapse of medial longitudinal arch, with eversion and abduction of the forefoot B. Exaggerated height of the medial longitudinal arch of the foot C. Collapse of long plantar ligament D. Collapse of deltoid ligament E. Collapse of plantar calcaneonavicular ligament

B. An injury to L4 would cause weakness in the patellar reflex and loss of cutaneous innervation to the medial side of the leg. The patellar reflex is used to test L2 to L4 nerve integrity. The motor side of the reflex is primarily derived from spinal nerves L2 and L3, whereas the sensory side of the arc is said to be principally from L4. The L4 spinal nerve supplies the L4 dermatome on the medial side of the leg and foot, by way of the saphenous nerve. It also supplies foot inversion, a function of the tibialis anterior and tibialis posterior muscles; the first is supplied by the deep fibular (peroneal) nerve, and the second supplied by the tibial nerve. Foot dorsiflexion is weakened because of partial denervation of the extensor digitorum longus, but L5 is still contributing to that function. The foot is everted because the S1-supplied (by the superficial fibular nerve) fibularis (peroneus) longus and brevis are unopposed. The Achilles reflex is also primarily supplied by S1. Hip movements are produced primarily by L5- and S1-supplied muscles, as is knee flexion.

A 55-year-old woman is admitted to the emergency department after an automobile crash. Physical examination reveals that the patient's foot is everted and she cannot invert it. A weakness in dorsiflexion and inversion of the foot is noted. Her ipsilateral patellar reflex is reduced in quality, although the Achilles tendon reflex is brisk. Knee extension is almost normal, as are all hip movements and knee flexion. Sensation is greatly reduced on the medial side of the leg. Which of the following nerves is most likely injured? A. Femoral nerve B. L4 spinal nerve C. L4 and L5 spinal nerves D. Common fibular (peroneal) nerve E. Tibial nerve

D. Injury to the dorsalis pedis artery on the dorsum of the foot can also cause trauma to the terminal portion of the deep fibular (peroneal) nerve. In the proximal part of the foot, this could result in loss of sensation between the first and second toes and paralysis of the extensor digitorum brevis and the extensor hallucis brevis muscles. In the distal part of the foot, only the sensory loss might be apparent. Clubfoot is a congenital malformation observed in pediatric patients. This syndrome combines plantar flexion, inversion, and adduction of the foot. Neither extension of the big toe by the extensor hallucis longus nor paralysis of the tibialis anterior (weakness of foot inversion) would occur by this injury because both of these muscles are innervated by the deep fibular (peroneal) nerve much more proximally in the leg

A 55-year-old woman is bitten by a dog in the dorsum of the foot and is admitted to the emergency department. The wound is cleaned thoroughly, during which it is seen that no tendons have been cut, but the dorsalis pedis artery and the accompanying nerve have been injured. Which of the following conditions would be expected during physical examination? A. Clubfoot B. Foot drop C. Inability to extend the big toe D. Numbness between the first and second toes E. Weakness in inversion of the foot

C. The deep fibular (peroneal) nerve passes deep to the extensor retinaculum and supplies the intrinsic muscles on the dorsum of the foot (extensors digitorum and hallucis longus) and the tarsal and tarsometatarsal joints. When it finally emerges as a cutaneous nerve, it is so far distal in the foot that only a small area of skin remains available for innervation: the web of skin between and contiguous sides of the first and second toes. The superficial fibular (peroneal) nerve supplies the skin on the anterolateral aspect of the leg and divides into the medial and intermediate dorsal cutaneous nerves, which continue across the ankle to supply most of the skin on the dorsum of the foot.

A 56-year-old diabetic man complains of repeated injury and ulcers to his right big toe. He also complains that he finds it difficult maintaining his shoes because the tips of the shoes around the toe area easily wear down. He also complains that for a while now, his first two toes "feel funny." He used to enjoy playing soccer on weekends but has found it difficult to be involved. Which of the following nerves is most likely affected? A. Superior gluteal nerve injury B. Inferior gluteal nerve injury C. Deep fibular (peroneal) nerve injury D. Superficial fibular (peroneal) nerve injury E. Common fibular (peroneal) nerve injury

B. The obturator nerve arises from the lumbar plexus and enters the thigh through the obturator canal. This nerve is responsible for innervation of the medial compartment of the thigh (adductor compartment). Injury to this nerve can result in weakened adduction and difficulty walking. The femoral nerve innervates muscles of the anterior compartment of the thigh that are responsible for hip flexion and leg extension. The sciatic nerve branches into the common fibular (peroneal) and tibial nerves. The common fibular (peroneal) nerve branches into the deep and superficial branches of the fibular (peroneal) nerve responsible for innervation of the anterior and lateral compartments of the leg, respectively. The tibial nerve innervates the muscles of the posterior compartment of the thigh and leg, which are responsible for extension of the hip, flexion of the leg, and plantar flexion of the foot.

A 56-year-old man with advanced bladder carcinoma suffers from difficulty while walking. Muscle testing reveals weakened adductors of the right thigh. Which nerve is most likely being compressed by the tumor to result in walking difficulty? A. Femoral B. Obturator C. Common fibular (peroneal) D. Tibial E. Sciatic

E. The lateral plantar artery provides origin to the deep plantar arterial arch. Medially, the vascular arch anastomoses with the distal portion of the dorsalis pedis by way of the deep plantar artery. The anterior tibial artery continues as the dorsalis pedis at the ankle joint. The fibular (peroneal) artery, by way of a perforating branch in some individuals, replaces the dorsal pedis. The arcuate artery, a branch of the dorsalis pedis, provides origin for the dorsal metatarsal arteries to the lateral toes.

A 58-year-old diabetic patient is admitted to the hospital with a painful foot. Physical examination reveals that the patient suffers from peripheral vascular disease. There is no detectable dorsalis pedis arterial pulse, but the posterior tibial pulse is strong. Which of the following arteries will most likely provide adequate collateral supply from the plantar surface to the toes and dorsum of the foot? A. Anterior tibial B. Fibular (peroneal) C. Arcuate D. Medial plantar E. Lateral plantar

A. The adductor hallucis muscle inserts upon the lateral side of the proximal phalanx of the great toe, and also the lateral sesamoid bone, by way of its oblique and transverse heads. It is supplied by the lateral plantar nerve. The abductor hallucis inserts upon the medial side of the proximal phalanx and the medial sesamoid bone of the great toe. The sesamoid bones are within the tendon of the flexor hallucis brevis and assist it in its function at the first metatarsophalangeal joint. The abductor and flexor hallucis brevis are innervated by the medial plantar nerve. The first dorsal interosseous muscle and the first lumbrical both insert on the medial side of the extensor mechanism of the second toe. The quadratus plantae arises from the calcaneus and inserts on the tendon of the flexor digitorum longus muscle. The first lumbrical is supplied by the medial plantar nerve. The quadratus plantae, the lumbricals 2 to 4, and all interossei are innervated by the lateral plantar nerve

A 58-year-old female dancer presented to the orthopedic clinic with a complaint of pain during her work because of bilateral bunions. She was referred to a podiatric surgeon who scheduled her for surgery. The protruding bony and soft tissues of the toe were excised, and a muscle was reflected from the lateral side of the proximal phalanx, together with a sesamoid bone, upon which the muscle also inserted. What muscle was this? A. Adductor hallucis B. Abductor hallucis C. First dorsal interosseous D. First lumbrical E. Quadratus plantae

A. Excessive compression of the prepatellar bursa, as in working on bended knees, can result in pain and swelling of the prepatellar bursa, the so called housemaid's knee. Prepatellar bursitis affects plumbers, carpet layers, and other people who spend a lot of time on their knees. The bursa normally enables the patella to move smoothly under the skin. The constant friction of these occupations irritates this small lubricating sac (bursa) located just in front of the patella, resulting in a deformable tense cushion of fluid. Treatment usually requires simple drainage, but this may need to be repeated and occasionally steroids introduced. Excessive irritation of the infrapatellar bursa in kneeling for frequent and long periods of time (as in prayer) can result in "parson's knee." The posterior cruciate ligament of the knee can be injured in sudden, strong flexion of the knee, with posterior displacement of the tibia upon the femur. The patellar retinacula are strong, tendinous bands of tissue that join the quadriceps tendon to the vastus lateralis and medialis muscles. The lateral meniscus is a cartilaginous structure between the lateral condyles of the femur and tibia.

A 58-year-old female employee of a housecleaning business visits the outpatient clinic with a complaint of a constant burning pain in her knees. Clinical examinations reveal a "housemaid's knee" condition (Fig. 5-9). Which of the following structures is most likely affected? A. Prepatellar bursa B. Infrapatellar bursa C. Posterior cruciate ligament D. Patellar retinacula E. Lateral meniscus

D. The farm instrument has injured the deep fibular (peroneal) branch of the common fibular (peroneal) nerve. It is vulnerable to injury as it arises from the common fibular (peroneal) at the neck of the fibula. The muscles denervated are largely dorsiflexors of the foot; hence, foot drop and a high stepping gait can occur. Sensation on the dorsum of the foot is still present; therefore, the superficial branch is mostly or entirely intact, although sensation between the first and second toes would be absent. Femoral nerve injury would result in loss of knee extension. Loss of the sciatic nerve would result in loss of both the tibial and common fibular (peroneal) nerves. Because plantar flexion is still functional, the tibial nerve has not been cut.

A 58-year-old male farmer was accidentally struck with a scythe (a long, curved cutting blade) by another worker while they were cutting wheat. He was admitted to the county hospital with severe bleeding. During physical examination the doctor noted that the patient had a foot drop; sensation was present over the dorsum of the foot and the skin of the posterior calf. Which of the following nerves was injured? A. Femoral nerve B. Sciatic nerve C. Superficial fibular (peroneal) nerve D. Deep fibular (peroneal) nerve E. Common fibular (peroneal) nerve

C. In piriformis entrapment, the sciatic nerve can be compressed when the piriformis is contracted, leading to painful sensations in the lower limb. These usually involve pain in the gluteal area, posterior thigh, and leg, most frequently resembling a disc lesion at L5-S1, with compression of the S1 spinal nerve. L4 compression would be rather unusual but would involve the quadriceps femoris knee extension, foot inversion, and sensory loss on the medial side of the leg. L5 compression would be indicated by weakness in hip abduction, knee flexion, and sensory loss on dorsal and plantar surfaces of the foot. S1 compression would weaken plantar flexion and foot eversion. Pudendal nerve entrapment would affect the perineal region. The fibular (peroneal) division of the sciatic nerve passes through the piriformis in some individuals, leading to L5, S1-S3 nerve compression.

A 58-year-old man is admitted to the hospital with pain in his lower limb for the past 2 months. Physical examination reveals point tenderness in the region of his greater sciatic foramen, with pain radiating down the posterior aspect of his thigh. An MRI examination reveals that the patient suffers from piriformis entrapment syndrome. He is directed to treatment by a physical therapist for stretching and relaxation of the muscle. Entrapment of which of the following nerves can mimic piriformis entrapment syndrome? A. L4 B. L5 C. S1 D. S2 E. S3

A. The patellar reflex causes extension of the knee and is produced by the quadriceps muscle group which consist of; biceps femoris and vastus lateralis, medialis and intermedius. Quadratus femoris is a lateral rotator of the thigh. The sartorius is a flexor of the hip and knee, and the pectineus is an adductor and flexor of the hip.

A 58-year-old man visited his physician for his annual check-up. Physical examination reveals a hyperreflexia patellar reflex. Which muscle(s) contribute(s) to the tendon that is struck when testing this reflex? A. Quadriceps femoris B. Quadratus femoris C. Sartorius D. Pectineus E. Biceps femoris

A. The saphenous nerve runs with the great saphenous nerve which was being removed from patient. Sensory innervation to the areas of loss described is by the L4 root, which is carried by the saphenous nerve. The obturator nerve innervates the skin on the superior medial thigh. Lateral femoral cutaneous innervates the lateral aspect of the thigh. The tibial nerve supplies cutaneous innervation to the lateral aspect of the leg and if damaged would also produce muscular dysfunction. The femoral nerve is a motor and sensory nerve and is the origin of the saphenous nerve.

A 58-year-old woman presents to the outpatient surgery clinic for removal of varicose veins on the medial aspect of her foot. The operation was successful however, one month later she reports loss of sensation over the medial aspect of her leg and foot. Which of the following nerves was most likely injured during the procedure? A. Saphenous B. Obturator C. Lateral femoral cutaneous D. Tibial E. Femoral

D. Duchenne muscular dystrophy is a condition that causes muscle weakness. It starts in childhood and may be noticed when a child has difficulty standing up, climbing, or running, which requires extension of the hip. This patient has the classic Gower's sign. The gluteus maximus functions primarily between the flexed and standing (straight) positions of the thigh, as when rising from the sitting position, straightening from the bending position, walking uphill and up stairs, and running.

A 6-year-old boy with a family history of muscular disease leading to wheelchair dependency in his maternal uncles presents with difficulty in standing from the seated position. He bends forward, uses his hands to help him push up from the floor, and then straightens his knees to stand. Which of the following muscles is most likely involved by this disease process? A. Tibialis posterior and gastrocnemius B. Quadratus femoris C. Gluteus medius and gluteus minimus D. Gluteus maximus E. Hamstrings F. Iliopsoas

B. The saphenous nerve is the longest and most widely distributed cutaneous branch of the femoral nerve; it is the only branch not from the sciatic nerve to extend beyond the knee. It gives sensory innervations to the medial aspect of the thigh, leg, and the medial planter arch. It accompanies the great saphenous vein over the medial side of the leg. The great saphenous vein is formed by the union of the dorsal vein of the great toe and the dorsal venous arch of the foot. It ascends anterior to the medial malleolus and passes posterior to the medial condyle of the femur and ends when it joins the femoral vein.

A 60-year-old man presents with pain on the medial aspect of his thigh. During physical examination he describes the pain to be constant, nonradiating and he also complains of numbness on the medial aspect of his leg and medial plantar arch. The nerve involved in this patient's numbness is closely associated with a structure with which of the following characteristics? A. Empties into the popliteal vein B. In its ascent in the medial aspect of the leg, it travels posterior to the medial condyle of the femur C. In its ascent in the medial aspect of the leg, it travels anterior to the medial condyle of the femur D. Arches posterior to the medial malleolus E. Is associated with nodes that drain to the horizontal group of inguinal nodes

E. Tarsal tunnel syndrome is a compression neuropathy resulting from the compression of the tibial nerve in the tarsal tunnel. The tarsal tunnel is located between the medial malleolus, the inferomedial surface of the calcaneus, and the flexor retinaculum. The contents are the tibial nerve and its plantar branches, the tendons of the tibialis posterior, flexor digitorum longus, and the flexor hallucis longus muscles together with the posterior tibial vessels. Any inflammation or swelling in the area will compress on these structures, most significantly the tibial nerve. The posterior tibial vein will be most easily compressed but the nerve is most clinically significant. Clinically, this syndrome is diagnosed with the patient's history and physical examination findings including a positive Tinel's sign (lightly tapping over the flexor retinaculum elicits numbness and tingling in the skin over the calcaneus and the sole of the foot).

A 60-year-old retired male marathon runner complains to his primary care physician that during his daily morning jog he experiences bouts of numbness and tingling on the medial aspect of his heel. Upon further examination the doctor discovers the patient has trouble tiptoeing and shows a positive Tinel's sign. Which of the following conditions is most characteristic of these symptoms? A. Plantar fasciitis B. Ankle inversion sprain C. Morton's neuroma D. Lateral ligament E. Tarsal tunnel syndrome

D. Spinal tuberculosis can spread within the sheath of the psoas major to its insertion with the iliacus upon the lesser trochanter, presenting there also with painful symptoms. The iliopsoas muscle is the principal flexor of the hip joint. Abduction of the hips is performed by the gluteus medius and minimus with assistance from short lateral rotator muscles. Extension of the hip is a function of the gluteus maximus, together with the hamstring muscles. Internal rotation is performed by the adductor muscle group.

A 61-year-old female immigrant had been diagnosed with spinal tuberculosis. The woman had developed a fluctuant, red, tender bulge on one flank, with a similar bulge in the groin on the same side. This presentation is likely due to spread of disease process within the fascia of a muscle with which of the following actions at the hip? A. Abduction B. Adduction C. Extension D. Flexion E. Internal rotation

D. The ankle jerk reflex, elicited by tapping the tendo Achilles with the reflex hammer, is mediated by the tibial nerve. The superficial fibular (peroneal) nerve supplies the foot evertor muscles of the lateral compartment of the leg and provides sensory supply for the dorsum of the foot. The deep fibular (peroneal) nerve innervates the foot extensor and invertor muscles in the anterior compartment of the leg and supplies skin between the first and second toes. The common fibular (peroneal) nerve combines the functions of the superficial and deep branches. The medial plantar nerve innervates the abductor and flexor muscles of the big toe, the first lumbrical muscle, and flexor digitorum brevis muscle and provides sensation for the medial plantar surface and three and a half toes. GAS

A 62-year-old man is admitted to the emergency department. Radiologic examination and the available data indicate the likelihood of a transient ischemic attack. During physical examination the ankle jerk reflex is absent. Which of the following nerves is most likely responsible for the reflex arc? A. Common fibular (peroneal) B. Superficial fibular (peroneal) C. Deep fibular (peroneal) D. Tibial E. Superficial and deep fibular (peroneal)

C. The patient's complaint is due to her case of hammertoes. Hammer toe can affect any toe but most commonly the second toe, then the third or fourth toes. It results most commonly from wearing shoes that are too short or shoes with heels that are too high. In hammertoe, the metatarsophalangeal joint is extended, the proximal interphalangeal joint is flexed, and the distal phalanx points downward, looking like a hammer. Hammertoe can occur as a result of a bunion. Calluses, or painful corns, can form on the dorsal surface of the joints. In claw toe, both the proximal and distal interphalangeal joints are strongly flexed, the result of muscle imbalance in the foot. Either hammertoe or claw toe can occur from arthritic changes. Pes cavus is the opposite of flat foot. In this case the patient has a high, flexed plantar arch; it occurs as a result of hereditary motor and sensory neural problems. It is painful because of metatarsal compression.

A 63-year-old woman visits the outpatient orthopedic clinic with the complaint of pain in her foot for more than a year. Radiologic and physical examinations give evidence of constant extension at the metatarsophalangeal joints, hyperflexion at the proximal interphalangeal joints, and extension of distal interphalangeal joints (Fig. 5-6). Which of the following terms is most accurate to describe the signs of physical examination? A. Pes planus B. Pes cavus C. Hammertoes D. Claw toes E. Hallux valgus

E. The posterior tibial artery provides most of the arterial supply for the neck and body of the talus bone. The fibular (peroneal) artery provides a small amount of vascular supply. The medial plantar and lateral plantar branches of the posterior tibial artery are distributed to tissues in the plantar surface of the foot. The dorsalis pedis is the continuation of the anterior tibial artery on the dorsum of the foot.

A 65-year-old man is admitted to the hospital after falling from his roof while cleaning leaves and pine needles from the gutters. Among other injuries suffered in his fall, radiologic examination reveals a fracture of the talus bone in one foot. Much of the blood supply of this bone can be lost in such an injury and can result in osteonecrosis. From what artery does this bone receive its primary vascular supply? A. Medial plantar B. Lateral plantar C. Dorsalis pedis D. Anterior tibial E. Posterior tibial

B. Generally, the angle of inclination between the neck and shaft of the femur in older age decreases to around 120 degrees. However, in pathologic conditions it can either increase or decrease from the predicted value. When the angle of inclination increases, it is referred to as coxa valga. Coxa vara on the other hand is a condition characterized by a decreased angle of inclination. Genu varum and genu valgum are deformities characterized by a decreased Q-angle and increased Q-angle, respectively. The Q-angle refers to the angle between the femur and tibia. Hallux valgus is a condition that presents with a lateral deviation of the large toe.

A 67-year-old woman has been suffering from osteoporosis for the past year. During her annual checkup, radiologic examination reveals an angle of 160 degrees made by the axis of the femoral neck to the axis of the femoral shaft. Which of the following conditions is associated with these examination findings? A. Coxa vara B. Coxa valga C. Genu valgum D. Genu varum E. Hallux valgus

A. The talocrural joint is a synovial hinge joint that connects the distal end of the tibia and fibula with the proximal end of the talus. The articulation between the tibia and the talus bears more weight than other joints. Dorsiflexion (toes pointing upward) and plantar flexion (toes pointing downward) are possible. Dorsiflexion is performed by the tibialis anterior, extensor hallucis longus, extensor digitorum longus, and peroneus tertius. Plantar flexion is performed by the gastrocnemius, soleus, plantaris, peroneus longus, peroneus brevis, tibialis posterior, flexor digitorum longus, and flexor hallucis longus. The movements of inversion and eversion take place at the talocalcaneal joint.

A 69-year-old woman, who fell down the stairs, presents to the emergency department. Radiologic imaging reveals a fracture of the talocrural (tibiotalar) joint. Which movements take place at this joint? A. Plantar flexion and dorsiflexion B. Inversion and eversion C. Plantar flexion, dorsiflexion, inversion, and eversion D. Plantar flexion and inversion E. Dorsiflexion and eversion

B. The medial plantar nerve innervates the abductor hallucis and both flexor hallucis longus and brevis. This nerve also provides motor supply for the flexor digitorum brevis and the first lumbrical. The lateral plantar nerve innervates all other intrinsic muscles in the plantar region of the foot. The sural nerve is sensory to the lateral posterior leg and lateral side of the foot; it arises from a combination of branches of the tibial nerve and common fibular (peroneal) nerve. The deep fibular (peroneal) nerve supplies dorsiflexors, toe extensors, and invertors of the foot.

A 7-year-old girl accidentally stepped on a sharp snail shell while walking to the beach. She was admitted to the hospital, where she received a tetanus shot, and the wound was cleaned thoroughly and sutured. One week later, during a return visit to her physician, it is seen that she has great difficulty in flexing her big toe, even though there is no inflammation present in the sole of the foot. Which nerve was most likely damaged by the piercing of the shell? A. Lateral plantar nerve B. Medial plantar nerve C. Sural nerve D. Superficial fibular (peroneal) nerve E. Deep fibular (peroneal) nerve

D. In claw toe, both the proximal and distal interphalangeal joints are strongly flexed, the result of muscle imbalance in the foot. With muscular imbalance, the extensors of the interphalangeal joints are overpowered by the long flexors. The metatarsophalangeal joint is extended, whereas in hammertoe it can be in a neutral position. Either hammer toe or claw toe can occur from arthritic changes. Hammertoe can affect any toe, but it most commonly affects the second toe, then the third or fourth toes. It results most commonly from wearing shoes that are too short or shoes with heels that are too high. In hammertoe, the metatarsophalangeal joint is extended, the proximal interphalangeal joint is flexed, and the distal phalanx may be dorsiflexed, or it may point downward, looking like a hammer. Hammertoe can occur as a result of a bunion. Calluses, or painful corns, can form on the dorsal surface of the joints. Hallux valgus is more commonly referred to as a bunion. The big toe is angulated toward the little toe and may override the second toe. The base of the first metatarsal bone is directed medially and is subject, painfully, to compression. Pes cavus is the opposite of flat foot; the patient has a high, flexed plantar arch. Pes cavus occurs as a result of hereditary motor and sensory neural problems. It is painful because of metatarsal compression.

A 71-year-old man is admitted to the orthopedic clinic with difficulties walking. The patient has a past history of polio. Physical and radiologic examinations reveal extension at the metatarsophalangeal joints with flexion of both the proximal and distal interphalangeal joints. Which of the following descriptions is most appropriate for this patient's condition? A. Hallux valgus B. Pes planus C. Hammertoes D. Claw toes E. Pes cavus

E. Hallux valgus, or lateral displacement of the great toe, usually presents as pain over the prominent metatarsal head, due to rubbing from shoes, and it can be associated with deformity of the second toe, which then tends to override the great toe. Hallux valgus is commonly known as bunion. In this deformity the big toe points toward the little toe; the base of the first metatarsal points medially, with a swollen bursal sac at the metatarsophalangeal joint. Excess bony growth of the distal protruding part of the metatarsal bone can also occur. Bunions occur only rarely in people who do not routinely wear shoes. Genu varus is also referred to as bowlegs, or bandy legs, in which the knees are bowed outward. The opposite of this is genu valgus, or knock knee. The normal angle between the femoral shaft and femoral neck is between 120 and 135 degrees. In coxa vara the angle between the shaft and neck of the femur is less than 120 degrees. This can result from fractures, other injuries, or congenital softness of the bone of the femoral neck. This defect results in limb shortening and limping. In coxa valga there is an increase in femoral shaft neck angulation, which can lead to hip subluxation or dislocation. Coxa valga results from weakness of the adductor musculature.

A 72-year-old man visits the outpatient clinic with a complaint of severe pain when walking. Physical examination reveals the problems in his feet as shown in Fig. 5-5. What is the most likely diagnosis? A. Coxa varus B. Coxa valgus C. Genu valgus D. Genu vara E. Hallux valgus

E. The ankle jerk reflex involves S1 and S2 levels. L2 to L4 are involved in the patellar reflex. L5 is not a component of a deep tendon reflex.

A 72-year-old woman is admitted to the emergency department after an episode of stroke. During neurologic examination the patient shows no response to the ankle reflex test. Which of the following nerve roots is responsible for this reflex? A. L2 B. L3 C. L4 D. L5 E. S1

B. The lateral circumflex femoral artery arises from the deep femoral (profunda femoris) artery of the thigh and sends a descending branch down the length of the femur to anastomose with the superior medial genicular artery and the superior lateral genicular artery. The medial circumflex femoral artery is responsible for supplying blood to the head and neck of the femur, and it does not anastomose with distal vessels at the knee. The first perforating artery sends an ascending branch that anastomoses with the medial circumflex femoral and the inferior gluteal artery in the buttock. The inferior gluteal artery is a branch of the internal iliac; it has important anastomotic supply to the hip joint. The typically small descending genicular branch of the femoral artery is given off just proximal to the continuation of the femoral artery as the popliteal.

A 72-year-old woman is admitted to the hospital with a painful right foot. A CT scan examination reveals a thrombotic occlusion of the femoral artery in the proximal part of the adductor canal. Which artery will most likely provide blood supply to the leg through the genicular anastomosis? A. Medial circumflex femoral B. Descending branch of the lateral circumflex femoral C. First perforating branch of the deep femoral D. Inferior gluteal E. Descending genicular branch of femoral

C. The iliofemoral ligament ("inverted Y ligament of Bigelow") is the most important ligament reinforcing the joint anteriorly that would resist both hyperextension and lateral rotation at the hip joint. The pubofemoral ligament reinforces the joint inferiorly and limits extension and abduction. The ischiofemoral ligament reinforces the joint posteriorly and limits extension and medial rotation. Negative pressure in the acetabular fossa has nothing to do with resisting hyperextension of the hip joint but does help resist dislocation of the head of the femur. The gluteus maximus muscle extends and laterally rotates the thigh and does not particularly resist hyperextension.

A 72-year-old woman suffered a hip dislocation when she fell down the steps to her garage. Which of the following is most significant in resisting hyperextension of the hip joint? A. Pubofemoral ligament B. Ischiofemoral ligament C. Iliofemoral ligament D. Negative pressure in the acetabular fossa E. Gluteus maximus muscle

E. In infants and children until about the age of 8 years, the head of the femur gets its arterial supply by a direct branch of the obturator artery (variably, the medial circumflex femoral). The arterial supply reaches the head of the femur at the fovea capitis by traveling along the ligament of the head of the femur. This source of supply is replaced later by vessels such as branches of the ascending branch of the medial circumflex femoral that pass into foramina of the neck of the femur within the capsule of the hip joint. Similar branches can arise from the lateral circumflex femoral and gluteal arteries. The deep circumflex iliac artery arises from the external iliac artery and supplies branches to the ilium, the iliacus muscle, and lower portions of the abdominal wall. The acetabular branch of the obturator artery often provides the branch to the head of the femur, an artery that normally regresses early in life, so that it supplies only the immediate area of the fovea capitis. The descending branch of the lateral circumflex femoral supplies the vastus lateralis muscle and participates in anastomoses at the knee. The second perforating branch of the deep femoral artery often supplies the nutrient artery of the shaft of the femur.

A 75-year-old man is admitted to the emergency department with severe pain at his right hip and thigh. An MRI examination reveals avascular necrosis of the femoral head. Which of the following arteries is most likely injured, resulting in avascular necrosis? A. Deep circumflex iliac B. Acetabular branch of obturator C. Descending branch of lateral circumflex femoral D. First perforating branch of profunda femoris E. Ascending branch of medial circumflex femoral

C. An intracapsular femoral neck fracture causes avascular necrosis of the femoral head because the fracture damages the radicular branches of the medial and lateral circumflex arteries that pass beneath the ischiofemoral ligament and pierce the femoral neck. Until an individual reaches about 6 to 10 years of age, blood supply to the head of the femur is provided by a branch of the obturator artery that runs with the ligament of the head of the femur. Thereafter, the artery of the ligament of the head of the femur is insignificant. Intertrochanteric fracture of the femur would not damage the blood supply to the head of the femur but would cause complications because the greater trochanter is an attachment site for several gluteal muscles. During childhood the obturator artery provides the artery of the ligament of the head of the femur. Thrombosis of the obturator artery could result in muscular symptoms, although there are several collateral sources of blood supply in the thigh. Comminuted fracture of the extracapsular femoral neck would not ordinarily imperil the vascular supply.

A 75-year-old man is transported to the emergency department with severe pain of his right hip and thigh. A radiologic examination reveals avascular necrosis of the femoral head (Fig. 5-2). Which of the following conditions most likely occurred to produce avascular necrosis in this patient? A. Dislocation of the hip with tearing of the ligament of the head of the femur B. Intertrochanteric fracture of the femur C. Intracapsular femoral neck fracture D. Thrombosis of the obturator artery E. Comminuted fracture of the extracapsular femoral neck

C. The medial circumflex femoral artery is responsible for supplying blood to the head and neck of the femur by a number of branches that pass under the edge of the ischiofemoral ligament. This artery is most likely at risk for injury in an extracapsular fracture of the femoral neck. The inferior gluteal artery arises from the internal iliac and enters the gluteal region through the greater sciatic foramen, below the piriformis. The first perforating artery sends an ascending branch that anastomoses with the inferior gluteal artery in the buttock. The obturator artery arises from the internal iliac artery and passes through the obturator foramen. It commonly supplies the artery within the ligament of the head of the femur but is not likely to be patent in a person of this age. The superior gluteal artery arises from the internal iliac artery and enters through the greater sciatic foramen above the piriformis.

A 75-year-old woman is admitted to the hospital after falling in her bathroom. Radiologic examination reveals an extracapsular fracture of the femoral neck. Which artery is most likely at risk for injury? A. Inferior gluteal B. First perforating branch of deep femoral C. Medial circumflex femoral D. Obturator E. Superior gluteal

A. A lateral blow to the knee could result in injury to three structures in the knee: anterior cruciate ligament, medial collateral ligament, and the medial meniscus. When all three structures are involved it is collectively called the "unhappy triad." Anterior drawer sign is due to injury to the anterior cruciate ligament and denoted by anterior displacement of the tibia in relation to femur, similar to pulling out a drawer from a desk.

A lateral blow to the knee during a tackle in a football game injures a 24-year-old woman. Field examination reveals an "anterior drawer sign." An MRI demonstrates injury to several structures of the knee, including her medial meniscus. Which structure might also have been injured by the tackle? A. Medial collateral ligament B. Lateral collateral ligament C. Lateral meniscus D. Posterior cruciate ligament E. Tendon of the semitendinosus

A. A lateral blow to the knee often produces a trio of injuries referred to as the "unhappy triad." This involves damage to the anterior cruciate ligament, medial meniscus, and medial collateral ligament. The medial meniscus and medial collateral ligament are often damaged together, as they are tightly attached to each other. The lateral collateral ligament and lateral meniscus would not be damaged because a blow to the lateral knee would not put strain on these structures. Damage to the posterior cruciate ligament would produce a positive "posterior drawer sign" and is typically damaged during a blow to the medial side of the knee. The posterior cruciate ligament is stronger than the anterior and is only typically damaged when a person falls on the tibial tuberosity of a flexed knee. Tendon of semitendinosus is on the medial side of the knee but is not attached closely to the other structures or taut in this injury type.

A lateral blow to the knee during a tackle in a football game injures a 24-year-old woman. Field examination reveals an "anterior drawer sign." An MRI demonstrates injury to several structures of the knee, including her medial meniscus. Which structure might also have been injured by the tackle? A. Medial collateral ligament B. Lateral collateral ligament C. Lateral meniscus D. Posterior cruciate ligament E. Tendon of the semitendinosus

A. The sartorius is indicated by the arrow in Figure 5-3. This muscle forms the roof of the subsartorial canal (Hunter's canal), with the adductor longus and vastus medialis forming other muscular borders. The femoral artery and vein, the saphenous nerve, the nerve to the vastus medialis, and the medial cutaneous nerve of the thigh all pass into this canal. The femoral artery leaves the canal by passing through the hiatus of the adductor magnus. The saphenous nerve emerges from the canal and from beneath the sartorius on the medial side of the lower limb proximally, thereafter providing sensory branches to the medial side of the lower limb and foot. Dr. Hunter mobilized the sartorius, thereby exposing the femoral artery (which continues as the popliteal artery beyond the adductor hiatus), which could be clamped while an aneurysmal popliteal artery was treated surgically.

A popliteal arterial aneurysm can be very fragile, bursting with great loss of blood and the potential loss of the leg if it is not dealt with safely and effectively. In the 18th century, Dr. John Hunter (1728-1793) discovered that if a primary artery of the thigh is temporarily compressed, blood flow in the popliteal artery can be reduced long enough to treat the aneurysm in the popliteal fossa surgically, with safety. What structure is indicated in Fig. 5-3 that is related to his surgical procedure? A. Sartorius B. Femoral vein C. Femoral artery D. Gracilis E. Adductor brevis

B. The obturator nerve is responsible for innervation of the thigh adductors which form the medial compartment of the thigh. The femoral nerve innervates the anterior compartment and is responsible for extension of the knee. Common fibular (peroneal) nerve supplies the anterior and lateral compartments of the leg while the tibial nerve supplies the posterior compartments of the leg and thigh. The common fibular (peroneal) and tibial nerves are branches of the sciatic nerve.

After a revascularization procedure involving the common iliac artery, a 68-year-old man has difficulty walking. Nerve conduction studies reveal decreased activity in the nerve that innervates the adductors of the thigh. Which nerve is this? A. Femoral B. Obturator C. Common fibular (peroneal) D. Tibial E. Sciatic

E. Plantar flexion is mostly due to the gastrocnemius and soleus muscles, which are supplied by the tibial nerve. The tibial nerve leaves the popliteal fossa by passing deep to the gastrocnemius and soleus muscles and lies posterior to the popliteal artery. Therefore a hematoma of the popliteal artery will also compress the nerve. Dorsiflexion of the foot is due to contraction of the muscles in the anterior compartment of the leg

After being struck from behind by a motor vehicle, a 55-year-old man presents to the hospital with a swelling of his right knee. Imaging reveals a large hematoma of the popliteal artery compressing his tibial nerve. Upon neurologic examination which movement would likely be diminished in strength? A. Dorsiflexion of the foot B. Flexion of the thigh C. Extension of the digits D. Extension of the leg E. Plantar flexion of the foot

D. The piriformis muscle arises from the pelvic surface of the sacrum, passes through the greater sciatic notch, and inserts at the greater trochanter. It is considered the "anatomical key" to gluteal anatomy; the greater sciatic foramen is the "door." The gluteus medius lies posterior to the piriformis. The sciatic nerve emerges from the greater sciatic foramen, normally through the infrapiriform's space. The spine of the ischium separates the greater and lesser sciatic foramina.

After dividing the overlying superficial tissues and gluteal musculature in a 68-year-old female patient, the orthopedic surgeon carefully identified the underlying structures while performing a total hip arthroplasty. The key landmark in the gluteal region, relied upon in surgical explorations of this area, is provided by which of the following structures? A. Gluteus medius B. Obturator internus tendon C. Sciatic nerve D. Piriformis muscle E. Spine of the ischium

C. The deep nodes are located beneath the deep fascia (fascia cribrosa) and lie along the medial side of the femoral vein. The presence of swollen inguinal lymph nodes is an important clinical sign because swelling may indicate an infection in the lower extremities. They then drain superiorly to the external iliac lymph nodes. The superficial nodes lie in the superficial fascia below the inguinal ligament and can be divided into horizontal and vertical groups. External iliac lies along the external iliac vessels; they are arranged in groups of three (anteriorly, medially, and lateral to vessels).

After suffering a deep stab wound to her posterior thigh, a 22-year-old woman presents to the emergency department. The wound is closed but the patient develops a subsequent wound infection. Which group of lymph nodes first receives drainage from this deep wound area, and would most likely be enlarged in this patient? A. External iliac B. Superficial inguinal C. Deep inguinal D. Common iliac E. Internal iliac

C. The fibularis (peroneus) brevis arises from the fibula and inserts upon the tuberosity at the base of the fifth metatarsal bone. Its attachment is often involved in an inversion fracture of the foot. This common fracture can often be overlooked when it is combined with an inversion sprain of the ankle. The fibularis (peroneus) longus arises from the fibula passes under the lateral malleolus, and then turns medially into the plantar surface of the foot, where it inserts upon the medial cuneiform and first metatarsal bones. The tibialis posterior arises from the tibia in the posterior compartment of the leg; it passes under the medial malleolus and inserts upon the navicular and metatarsal bones. The extensor digitorum brevis arises dorsally from the calcaneus and inserts upon the proximal phalanges of the lateral toes. The adductor hallucis arises from the lateral metatarsals and transverse tarsal ligament and inserts upon the proximal phalanx and lateral sesamoid bone of the big toe.

An 18-year-old professional tennis player fell when she leaped for an overhead shot and landed with her foot inverted. Radiologic examination in the hospital revealed an avulsion fracture of the tuberosity of the fifth metatarsal. Part of the tuberosity is pulled off, producing pain and edema. Which of the following muscles is pulling on the fractured fragment? A. Fibularis (peroneus) longus B. Tibialis posterior C. Fibularis (peroneus) brevis D. Extensor digitorum brevis E. Adductor hallucis

A. The patient has bowlegs, or genu varus. The opposite of this is genu valgus, or knock knee. The normal angle between the femoral shaft and femoral neck is between 120 and 135 degrees. In coxa vara the angle between the shaft and neck of the femur is less than 120 degrees. This can result from fractures, other injuries, or congenital softness of the bone of the femoral neck. This defect results in limb shortening and limping. In coxa valga there is an increase in femoral shaft neck angulation, which can lead to hip subluxation or dislocation. Coxa valga results from weakness of the adductor musculature. Hallux valgus is commonly known as bunion. In this deformity the big toe points toward the little toe and may override the second toe; the base of the first metatarsal points medially, with a swollen bursal sac at the metatarsophalangeal joint. Excess bony growth of the distal protruding part of the metatarsal bone can also occur. Bunions occur only rarely in people who do not routinely wear shoes.

An 81-year-old man is admitted to the emergency department with severe pain in his knees. The patient has a long history of osteoarthritis. Radiologic examination reveals degeneration of the joints of his lower limbs. The degeneration is more severe on the medial side of the knees, which causes his knees to be bowed outward when he stands upright. Which of the following terms best describes the condition of his knees? A. Genu varus B. Genu valgus C. Coxa varus D. Coxa valgus E. Hallux valgus

C. The obturator artery provides the artery within the ligament of the head of the femur (in about 60% of cases), the artery that supplies the head of the femur, primarily during childhood, later becoming atretic. In the adult this artery supplies only the area of the fovea of the head of the femur. The ligament of the head of the femur arises from the acetabular notch, thereafter receiving the little artery. In some individuals the medial circumflex femoral gives origin to the artery of the head. In the adult the arterial supply of the neck and head is provided by intracapsular branches of the medial circumflex femoral and lateral circumflex femoral arteries that pierce the neck of the femur, with some supply also from the gluteal arteries. The lateral circumflex femoral artery arises from the deep femoral and supplies the vastus lateralis. The pudendal artery arises from the internal iliac and provides blood supply for the structures of the perineum. Quite often, when an older patient with osteoporosis has a hip fracture, the femoral neck may have fractured, precipitating a fall, rather than the fall resulting in the hip fracture.

An 82-year-old grandmother slipped on the polished floor in her front hall and was transported to the emergency department and admitted for examination with a complaint of great pain in her right lower limb. During physical examination it is observed by the resident that the right lower limb is laterally rotated and noticeably shorter than her left limb. Radiologic examination reveals an intracapsular fracture of the femoral neck. Which of the following arteries supplies the head of the femur in early childhood but no longer in a patient of this age? A. Superior gluteal B. Lateral circumflex femoral C. A branch of the obturator artery D. Inferior gluteal E. Internal pudendal


Set pelajaran terkait

Chapter V: Standard English - Grammar, Punctuation, & Spelling

View Set

NUR 1018 - Gas exchange - adaptive Q's

View Set

Rosetta Stone French Unit 5 Lessons 1-3

View Set